Differential Equations 1
Differential Equations 1
• ILO 1- Identify, define and classify different terms, concepts, conditions, forms and
uses of a differential equation.
• ILO 3- Analyze the real world applications and model them as differential equation/s.
Online Quiz 1 4%
Online Quiz 2 10%
Online Quiz 3 8%
Online Quiz 4 4%
Online Quiz 5 4%
Online Quiz 6 8%
Online Quiz 7 8%
Online Quiz 8 4%
Midterm Examination 25%
Final Examination 25%
Module 1 Introductory Topics
Introduction
In this module, we will discuss several introductory topics that are essential to the study
of differential equation. Several definitions and other necessary skills in solving differential
equations are discussed in this module. Student will learn different forms and structure of
a differential equation. Also, they will learn to create a differential equation based on a
problem as well as finding solutions of initial-value problem differential equation.
Topic Outcomes
5. Show the solution of an initial-value problem by imposing the initial condition to get
the value of an arbitrary constant.
6. Show all the intervals in which the solution of a given DE is unique through partial
derivative correctly.
Definitions and Terms
Many of the principles, or laws, underlying the behavior of the natural world are
statements or relations involving rates at which things happen. When expressed in
mathematical terms the relations are equations and the rates are derivatives.
When an equation involves one or more derivatives with respect to a particular variable,
that variable is called an independent variable. A variable is called dependent if the derivative
of that variable occurs. In equation (5); i is the dependent variable, t the independent
variable and L, R, C, E, and ω are called parameters.
Differential Equation Defined
An equation containing the derivatives of one or more dependent variables, with respect
to one or more independent variables, is said to be a differential equation (DE).
Classifications of DE in terms of
• Type
• Order
• Linearity
Classifications by Type
Classifications by Order
The order of a differential equation (either ODE or PDE) is the order of the highest
derivative in the equation. For example, equation (1) is a first-order equation while equation
(2) is a second-order equation. First-order differential equations are written in the form
M (x, y)dx + N (x, y)dy = 0 for convenience.
In symbols, one can express an nth-order differential equation in one variable by the
general form
F x, y, y 0 , . . . , y (n) = 0
(12)
where F is areal-valued function of n + 2 variable: x, y, y 0 , . . . , y (n) .For both practical and
theoretical reasons it is assumed that it is possible to solve an ordinary differential equation
in the form (12) uniquely for the highest derivative of y (n) in the term of the remaining n + 1
variables. The differential equation:
dn y 0 (n)
= f x, y, y , . . . , y (13)
dxn
where f is a real-valued continuous function, is referred to as the normal form of equation
(12).
Classifications by Linearity
or
dn y d(n−1) y dy
an (x)
n
+ a n−1 (x) (n−1)
+ · · · + a1 (x) + a0 (x)y = g(x) (14)
dx dx dx
Two important cases of (14) are first-order differential equation (n=1) and second-order
differential equation (n=2):
dy
a1 (x) + a0 (x)y = g(x)
dx
and
d2 y dy
a1 (x) 2 + a1 (x) + a0 (x)y = g(x) (15)
dx dx
In the additive combination on the left-side of (14) it can be seen that the characteristic two
properties of a linear ordinary differential equations are as follows:
• the dependent variable y and all its derivatives y 0 , y 00 , . . . , y (n−1) are of the first degree
The equations
(y − x) dx + 4x dy = 0
y 00 + 2y 0 + y = 0
and
d3 y dy
3
+ x − 5y = ex
dx dx
are, in turn, first-order linear differential equations. Note that the first equation can be
rewritten as 4xy 0 + y = x, thus it is linear in the variable y.
A non-linear ordinary differential equation is simply one that is not linear. Non-linear
functions of the dependent variable, or its derivatives, such as sin y or ey , cannot appear in
a linear equation.
for all x in I.
Interval of Definition
1 4 dy 1
Therefore, y = x is a solution of = xy 2 on the interval (−∞, ∞).
16 dx
Example 2. Verifying a solution of an Equation
Solution:
Substitute the function to the equation, then compare the left-hand and the right-hand side.
Left-hand side:
Right-hand side:
0
Therefore, y = xex is a solution of y 00 − 2y 0 + y = 0 on the interval (−∞, ∞).
Note that in this example, the differential equation has a constant solution y = 0 on
the interval −∞ < x < ∞. A solution of a differential equation that is identically zero on
an interval I is said to be a trivial solution.
Elimination of Arbitrary Constants
There are several ways a differential equation may come up, some will be discussed in
modeling. There is one way of obtaining a differential equation by stating some families of
relations defined by a parameter/s called arbitrary constants. These represent any random
value; however it is not a variable although it could take in any value but once it is given a
value it would no longer be given any more values like in the case of a variable.
Methods of elimination may vary with the way in which a constant is placed in a
relation. Methods that is effective with one problem may be poor for other problems.
Example 1.
x3 − 3x2 y =c
dy
3x2 − 3x2 − 6xy =0
dx
dy
−3x2 = 6xy − 3x2
dx
dy
x = (2y − x)
dx
xdy = (2y − x)dx
−(2y − x)dx + xdy =0
(2y − x)dx − xdy =0
Example 2.
y sin x − xy 2 =c
dy dy
y cos x + sin x − 2xy − y2 =0
dx dx
dy dy
sin x − 2xy = y 2 − y cos x
dx dx
dy
(sin x − 2xy) = y(y − cos x)
dx
(sin x − 2xy)dy = y(y − cos x)dx
−y(y − cos x)dx + (sin x − 2xy)dy =0
y(y − cos x)dx − (sin x − 2xy)dy =0
Example 3.
dy
x2 + 2xy = c (1)
dx
Since c 6= 0, solving c from the main relation we have,
x2 y = 1 + cx
x2 y − 1 = cx
x2 y − 1
c=
x
then substitute this c to (1),
dy x2 y − 1
x2 + 2xy =
dx x
Simplifying,
dy
x3 + 2x2 y = x2 y − 1
dx
dy
x3 = x2 y − 2x2 y − 1
dx
dy
x3 = −x2 y − 1
dx
3
x dy = −(x2 y + 1)dx
(x y + 1)dx + x3 dy
2
=0
Example 4.
x = c1 cos ωt + c2 sin ω
Solution:
Since there are two arbitrary constants to be eliminated, obtain the first and the second
derivatives,
dx
= −c1 ω sin ωt + c2 ω cos ωt
dt
d2 x
2
= −c1 ω 2 cos ωt − c2 ω 2 sin ωt
dt
d2 x
= −ω 2 (c1 cos ωt + c2 sin ωt)
dt2
Notice that, c1 cos ωt+c2 sin ωt is simply equal to x from the given relation. Then the second
derivative can be expressed as
d2 x
= −ω 2 x
dt2
or
d2 x
+ ω2x = 0
dt2
Example 5.
y2
4a =
x
dy y2
2y =
dx x
2xydy = y 2 dx
2xydy − y 2 dx = 0
Example 6.
y = c1 + c2 e3x
Solution:
Obtain the first and the second derivatives of the given relation,
y = c1 + c2 e3x
dy
= 3c2 e3x
dx
d2 y
= 3(3c2 e3x )
dx2
d2 y dy
2
=3
dx dx
2
dy dy
2
−3 =0
dx dx
or
y 00 − 3y 0 = 0
Example 7.
y = x + c1 ex + c2 e−x
Solution:
Since there are two arbitrary constants to be eliminated, obtain the first and the second
derivatives,
y 0 = 1 + c1 ex − c2 e−x
y 00 = c1 ex + c2 e−x (1)
From the given relation,
y = x + c1 ex + c2 e−x
y − x = c1 ex + c2 e−x
Substituting to (1),
y 00 = c1 ex + c2 e−x
y 00 = y − x
or
y 00 − y + x = 0
Example 8.
y = x2 + c1 e2x + c2 e3x
Solution:
y 00 − 5y 0 + 6y = 6x2 − 10x + 2
Example 9.
y = Ae2x + Bxe2x
Solution:
2y − y 0 = −Be2x (4)
2y 0 − y 00 = −2Be2x (5)
y 00 − 4y 0 + 4y = 0
Families of Curves
General Form: Ax + By + C = 0
Point-Slope Form: y − y1 = m (x − x1 )
Slope-Intercept Form: y = mx + b
y2 − y1
Two-Point Form: y − y1 = (x − x1 )
x2 − x1
x y
Intercept Form: + =1
a b
Normal Form: x cos α + y sin α = ρ
Solution:
Some members of the family are drawn first.
As seen from the sketch, the lines have a common y-intercept b = 0 and have different slopes,
we can use the slope-intercept form.
y = mx + b
Since b = 0, we have
y = mx
which has one arbitrary constant m.
To eliminate m, we need to find the only the first derivative.
y = mx
Find the differential equation representing all lines with a slope 2/3.
Solution:
Some members of the family are drawn first.
As seen from the sketch, there is only one parameter that is changing and it is the y-intercept.
We can use the slope-intercept form,
2
y= +b
3
Differentiating,
2
y0 =
3
Example 3.
Find the differential equation representing all circles with center at the origin.
Solution:
The equation to use is
x2 + y 2 = r 2
wherein it only has one arbitrary constant r.
2x dx + 2y dy = 0
Rearranging,
x dx + y dy = 0
Example 4.
Find the differential equation representing all parabolas with vertex at the origin and focus
located on the y-axis.
Solution:
Some members of the family are drawn first.
It can be seen that the parabolas have the y-axis to be the principal axis, thus the family’s
equation is
(x − h)2 = 4a (y − k)
wherein, the vertex is the origin then h = 0 and k = 0. The equation to use now is
x2 = 4ay
To eliminate a, we need to do one differentiation.
x2 = 4ay (6)
Get the first derivative
4ay 0 = 2x
Solve for 4a
2x
4a =
y0
Substitute 4a in (1),
2x
x2 = ·y
y0
Simplifying,
x0 − 2y = 0
Example 5.
Find the differential equation representing all circles with center at the x-axis.
Solution:
Some members of the family are drawn.
From the figure, it can be seen that the center may have the coordinates (h, k) wherein h
could be any value and k = 0. The radii of the circles are also varying.
Hence the equation to use is
(x − h)2 + y 2 = r2 (1)
which has two arbitrary constants.
First differentiation,
2 (x − h) + 2yy 0 = 0
Simplify,
yy 0 + x = h
Second differentiation,
2
yy 00 + (y 0 ) + 1 = 0
Example 6.
Find the differential equation representing ellipses and hyperbolas with axes on the
coordinate axes.
Solution:
We can write the equation as
ax2 + by 2 = 1. (1)
x(y 0 )2 − yy 0 + xyy 00 = 0
Example 7.
Find the differential equation representing all circles with center on the line y = x and
passing through the origin.
Solution:
Some members of the families are drawn.
x2 + 2xy − y 2 dx + y 2 + 2xy − x2 dy = 0
Example 8.
Solution:
We can use the standard forms of the equation of a line with the least number of arbitrary
constant. So we will use the slope-intercept form.
y = mx + b
First derivative,
y0 = m
Second derivative,
y 00 = 0
No more arbitrary constant.
Example 9.
Find the differential equation of all sine waves with a period of 2π and a phase shift of 0◦ .
Solution:
Some members of the families are drawn.
y = A sin x
Solve:
dn y
= f x, y, y 0 , y 00 , . . . , y (n−1)
n
(1)
dx
Subject to:
has at least two solutions. As illustrated in Figure (4), the graphs of both passes throught
the point (0, 0).
1
Figure 4: Solutions of y 0 = xy 2 subject to the initial condition y(0) = 0.
Bibliography
[Boyce and Diprima, 2001] Boyce, W. and Diprima, R. (2001). Elementary Differential
Equations with Boundary Value Problems. John Wylie & Sons, Inc.
[Bronson and Costa, 2009] Bronson, R. and Costa, G. (2009). Schaum’s Outline of
Differential Equations. McGraw-Hill Education, 3rd edition.
[Lebl, 2019] Lebl, J. (2019). Notes on Diffy Qs: Differential Equations for Engineers.
Independently Published.
[Rainville et al., 2013] Rainville, D., Bedient, P., and Bedient, R. (2013). Elementary
Differential Equations: Pearson New International Edition. Pearson Education, Limited.
[Zill, 2012] Zill, D. (2012). A First Course in Differential Equations with Modeling
Applications. Cengage Learning.
Module 2 First-Order Differential
Equations
Introduction
In this module, we will discuss several methods to find solutions of first-order differential
equations. After finishing this module, students are expected to be familiar with the different
forms of a first-order differential equation and how to find solutions for them.
Topic Outcomes
2. Show the solution of a variable separable DE using the correct method, expressed in
its simplest form.
4. Show the solution of a DE with homogeneous coefficients using the correct method,
expressed in its simplest form.
6. Show the solution of an exact DE using the correct method, expressed in its simplest
form.
8. Show the solution of a linear DE using the correct method, expressed in its simplest
form.
9. Recognize that a 1st order DE has an integrating factor by grouping terms to give
exact correct differentials.
10. Show solution of a DE by using integrating factors, expressed in its simplest form.
11. Recognize that a non-exact DE can be made exact by means of the use of a correct
integrating factor.
13. Recognize that terms in a DE can be used to make substitution by making appropriate
substitution.
14. Show solutions of a DE by making substitutions out of its terms to make use of other
methods.
16. Show the solution of a Bernoulli DE using the correct method, expressed in its simplest
form.
17. Recognize that coefficients in a DE are linear in two variables by finding the intersection
of the lines or making appropriate substitution correctly.
18. Show the solution of a DE with coefficients linear in two variables using the correct
method, expressed in its simplest form.
Separation of Variables
then the equation is said be variable separable. The solution can be immediately obtained
by integrating both sides of (2).
Z Z
A (x) dx + B (y) dy = c
Another definition:
A first-order differential equation in the form
dy
= g(x)h(y)
dx
is said to have separable variable.
Example 1.
dy
(1 − x) = y2
dx
multiplying both sides by dx,
(1 − x)dy = y 2 dx
dy dx
=
y2 1−x
Integrating both sides of the equation gives,
Z Z
dy dx
2
=
y 1−x
1
− = − ln |1 − x| + c
y
1
− = − ln |1 − x| + ln c
y
Simplifying gives the final result as
y ln |c(1 − x)| = 1
Example 2.
xdx dy
+ 3 =0
ex2 y
2 dy
xe−x dx + 3 = 0
y
Integrating both sides of the equation,
Z Z
−x2 dy
xe dx + =c
y3
1 2 1
− e−x − y −2 = c
2 2
2
e−x + y −2 = −2c
2
e−x + y −2 = c
Example 4.
1 1
(2y dx) = (3x dy)
xy xy
2 dx 3 dy
=
x y
Integrating both sides,
2 dx 3 dy
=
x y
Z Z
dx dy
2 =3 +c
x y
2 ln |x| = 3 ln |y| + ln |c|
ln |x2 | = ln |y 3 | + ln |c|
ln |x2 | = ln |cy 3 |
x2 = cy 3
Example 5.
1 2x
2x
1
2x
ye dx = (4 + e )dy
y(4 + e ) y(4 + e2x )
e2x dx dy
2x
=
4+e y
Integrating both sides, we have
e2x dx
Z Z
dy
= +c
4 + e2x y
1
ln |4 + e2x | = ln |y| + ln |c|
2
ln |4 + e2x | = 2 ln |y| + 2 ln |c|
ln |4 + e2x | = ln |y 2 | + ln |c2 |
ln |4 + e2x | = ln |c2 y 2 |
4 + e2x = c2 y 2
Notice that c2 can be written as just c, since they are both arbitrary, but we should
note that the left side of the equation is always greater than 0. Therefore, if we just write c,
we will be restricted to values of c ≥ 0.
Example 6.
dr = b cos θ dr + br sin θ dθ
dr − b cos θ dr = br sin θ dθ
(1 − b cos θ) dr = br sin θ dθ
dr b sin θdθ
=
r 1 − b cos θ
Integrating both sides of the equation,
Z Z
dr b sin θdθ
= +c
r 1 − b cos θ
ln |r| = ln |1 − b cos θ| + ln |c|
ln |r| = ln |c(1 − b cos θ)|
r = c(1 − b cos θ)
Example 7.
x dx dy
− =0
x+2 y
xdx (y + 1)dy
− =0
e−x y3
xex dx − y −3 (y + 1)dy = 0
Solve the particular solution given the initial condition equation 2xyy 0 = 1 + y 2 , when x = 2
and y = 3.
Solution:
Divide the given equation by x(1 + y 2 ),
2xyy 0 = 1 + y 2
dy
2xy = 1 + y2
dx
2y dy dx
2
=
1+y x
Integrate both sides of equation
Z Z
2ydy dx
2
=
1+y x
2
ln |1 + y | = ln |x| + ln |c|
ln |1 + y 2 | = ln c|x|
1 + y 2 = cx
1 + y 2 = cx
1 + 32 = c(2)
c=5
1 + y 2 = 5x
Equations with Homogeneous Coefficients
Polynomials in which all the terms are of the same degree, such as
x2 + 2xy + y 2 x3 + y 3 x4 y + 8y 5
are called homogeneous polynomials.
Let us extend the concept of homogeneity and define what a homogeneous function is.
Definition
The function f (x, y) is said to be homogeneous of degree k in x and y if, and only if,
f (λx, λy) = λk f (x, y)
Theorem 0.0.1. If M (x, y) and N (x, y) are both homogeneous and of the same degree, then
M (x, y)
the function is homogeneous of degree zero.
N (x, y)
Example 1.
Determine if the function f (x, y) is homogeneous and if it is, state the degree of the function.
y x4
f (x, y) = 2y 3 e x −
x + 3y
Solution:
Original function
y x4
f (x, y) = 2y 3 e x −
x + 3y
f (λx, λy),
(λx)4 x4
λy y
3 3 3 x
f (λx, λy) = 2(λy) e λx − = λ · 2y e −
λx + 3λy x + 3y
Now,
f (λx, λy) = λ3 f (x, y)
Therefore, f (x, y) is homogeneous of degree 3.
Equations with Homogeneous Coefficients
Suppose the coefficients M and N in the differential equation of the first order
M (x, y) + N (x, y) = 0
are both homogeneous and are of same degree in x and y. We can put the equation in the
form
dy M (x, y)
+ = 0.
dx N (x, y)
By Theorems (1) and (2), we can put it in the form
dy y
+f =0
dx x
From this we may introduce a new variable v such that y = vx. Then the equation
becomes
dv
x + v + f (v) = 0
dx
in which the variable are now separable.
The method would have been equally successful if we used x = vy to obtain an equation
in y and v. However, it is sometimes easier to substitute for the variable whose differential
has the simpler coefficients.
Example 2.
Solve the equation
2(2x2 + y 2 )dx − xydy = 0
Solution:
From M dx + N dy = 0, both M and N are homogeneous functions of degree 2, let us put
y = vx
dy = vdx + xdv
Substitute to the original equation,
2(2x2 + v 2 x2 )dx − x2 v(vdx + xdv) = 0
(4x2 + 2v 2 x2 − v 2 x2 )dx − vx3 dv = 0
(4x2 + v 2 x2 )dx − vx3 dv = 0
x2 (4 + v 2 )dx − vx3 dv = 0
By separation of variables, divide the equation by (4 + v 2 )x3 ,
dx vdv
− =0
Z x Z 4 + v2
dx vdv
− =c
x 4 + v2
1
ln |x| − ln (4 + v 2 ) = ln |c|
2
2 ln |x| − ln (v 2 + 4) = ln |c|
2
x
ln 2 = ln |c|
v + 4
2
ln x 2
e 4+v = eln |c|
x2
=c
4 + v2
x2 = c(4 + v 2 )
y
Since v = ,
x
2
2 y
x =c 4+
x
2
y
x2 =c 4+
x2
2
4x + y 2
x2 =c
x2
x4 = c(4x2 + y 2 )
Example 3.
x = vy
dx = vdy + ydv
x
Since v = , then
y
2
x2
6 ln |y| − = ln |c|
y2
6y 2 ln |y| − x2 = y 2 ln |c|
6y 2 ln |y| − y 2 ln |c| = x2
y 2 (ln (y 6 ) − ln |c|) = x2
6
y
x2 = y 2 ln
c
x = y ln |cy 6 |
2 2
Example 4.
y
Since v = , then
x
x
cos v = ln
c
y x
cos = ln
x c
y
cos = ln |cx|
x
Example 5.
Solution:
Since M and N are homogeneous function of degree 1, then let
y = vx
dy = vdx + xdv
xdx + x2 ln |v|dv = 0
dx
+ ln |v| = 0
Z xZ
dx
+ ln |v| = 0
x
ln |x| + v ln |v| − v = ln |c|
y
Since v = , then
x
ln |x| + v ln |v| − v = ln |c|
y y y
ln |x| + ln − = ln |c|
x x x
y
x ln |x| + y ln − y = cx
x
x ln |x| + y ln |y| − y ln |x| − y = cx
(x − y) ln |x| + y ln |y| = cx + y
Example 6.
Solve the equation
y 2 dy = x(xdy − ydx)ex/y
Solution:
Express the given equation as
y 2 dy = x(xdy − ydx)ex/y
(y 2 − x2 ex/y )dy = −xyex/y dx
Since both coefficients are homogeneous and of degree 2, let
x
x = vy; v=
y
dx = vdy + ydv
x
Since v = , then
y
ln |y| − ln |c| = ev (v − 1)
y x/y x
ln = e
−1
c y
x/y x − y
y
ln = e
c y
y
y ln = (x − y)ex/y
c
y ln |cy| = (x − y)ex/y
Example 7.
Solve the equation
2y
xdx + sin (ydx − xdy) = 0
x
Solution:
Rewrite the given equation as
2 y 2 y
x + y sin dx − x sin dy = 0
x x
Let
y
y = vx; v=
x
dy = vdx + xdv
x + vx sin v dx − x sin2 v(vdx + xdv) = 0
2
x2 (v + 1)2 = c(v − 2)
y
Since v = , then
x
y 2 y
x2 +1 =c −2
x x
2
x(y + x) = c(y − 2x)
Example 9.
Solution:
Let
y
y = vx; v=
x
dy = vdx + xdv
2vt3 dt + t4 (v 2 + 1)dv = 0
dt v 2 + 1
2 + dv = 0
t v
dt dv
2 + vdv + =0
t v
1
2 ln |t| + v 2 + ln |v| = ln |c|
2
s
Since v = , then
t
2
1 s s
2 ln |t| + + ln = ln |c|
2 t t
2
1 s
2 ln |t| + + ln |s| − ln |t| = ln |c|
2 t2
s2
ln |s| + ln |t| − ln |c| + 2 = 0
2t2
st s
ln + 2 = 0
c 2t
st
2t2 ln + s2 = 0
c
2 2
st
s = −2t ln
c
2 2
s = −2t ln |cst|
Exact Equations
∂f ∂f
dx + dy = 0 (2)
∂x ∂y
Definition
A differential expression M (x, y)dx + N (x, y)dy is an exact differential in a region R of
the xy-plane if it corresponds to the differential of some function f (x, y) defined in R. A
first-order differential equation in the form
is said to be an exact equation if the expression on the left-hand side is an exact differential.
∂M ∂N
=
∂y ∂x
Method of Solution
Given an equation M (x, y)dx + N (x, y)dy = 0, which is exact. Then there exist a function
F (x, y) for which
∂F
= M (x, y)
∂x
Solving for F (x, y), Z
F (x, y) = M (x, y)dx + T (y)
∂F
where the arbitrary function T (y) is the constant of integration. And assuming = N (x, y)
∂y
Z
∂
M (x, y) dx + T 0 (y) = N (x, y)
∂y
Giving us Z
0 ∂
T (y) = N (x, y) − M (x, y) dx
∂y
Finally integrating to find T (y) with respect to y.
F (x, y) = c
and Z
0 ∂
T (x) = M (x, y) − N (x, y) dy
∂x
The implicit solution is still
F (x, y) = c
Example 1.
Solve the equation
(x + 2y)dx + (2x + y)dy = 0
Solution:
Test for exactness,
∂M ∂N
= 2; =2
∂y ∂x
∂M ∂N
Since = , therefore the equation is exact. Hence, its solution is F = c, where
∂y ∂x
∂F ∂F
= M; =N
∂x ∂y
Using
∂F
= M = x + 2y
∂x
Solving for F by integrating both sides with respect to x, holding y as constant,
Z Z
∂F = (x + 2y)∂x
x2
F = + 2xy + T (y)
2
To solve for T (x), find the derivative of F with respect to y
∂F
= 2x + T 0 (y)
∂y
∂F
Since = N , then
∂y
2x + T 0 (y) = 2x + y
T 0 (y) = y
Z Z
0
T (y)dy = ydy
y2
T (y) =
2
Therefore,
x2 y2
F = + 2xy +
2 2
And the set of solutions of the given equation is defined by
x2 y2
+ 2xy + =c or x2 + 4xy + y 2 = c
2 2
Example 2.
∂F = x2 + 2y∂y
2y 2
F = x2 y + + T (x)
2
F = x2 y + y 2 + T (x)
∂F
Solve for T (x), find the derivative of F with respect to x that is, = 2xy − T 0 (x)
∂x
∂F
Since = N then,
∂x
2xy + T 0 (x) = 2xy − 3x2
T 0 (x) = −3x2
T (x) = −x3
x2 y + y 2 − x3 = C
Example 3.
Solve the equation
Solution:
∂M ∂N
= −2 sin (2y) − 6x2 y = −2 sin (2y) − 6x2 y
∂y ∂x
Hence, the equation is exact and its solution is F = c, where
∂F
= cos (2y) − 3x2 y 2
∂x
and
∂F
= cos (2y) − 2x sin (2y) − 2x3 y
∂y
Solving for F ,
∂F
= cos (2y) − 3x2 y 2
∂x
∂F = cos (2y) − 3x2 y 2 ∂x
∂F
= x2 − x sec2 y + T 0 (y)
∂y
∂F
Since = N , then
∂y
Therefore,
F = x2 y − x tan y + c1
and the set of solutions of the given equation is defined by
x2 y − x tan y = c
Example 5.
Solve the equation
cos x cos y − cot x dx − sin x sin ydy = 0
Solution:
∂M ∂N
= −x cos x sin y = −x cos x sin y
∂y ∂x
Hence, the equation is exact and its solution is F = c, where
∂F ∂F
= cos x cos y − cot x and = − sin x sin y
∂x ∂y
Solving for F using
∂F
=N
∂y
we have
∂F
= − sin x sin y
∂y
∂F = − sin x sin y∂y
F = sin x cos y + T (x)
∂F
Solving for T (y) find , we have
∂x
∂F
= cos x cos y + T 0 (x)
∂x
∂F
Since = M , then
∂x
cos x cos y + T 0 (x) = cos x cos y − cot x
T 0 (x) = − cot x
T (x) = − ln | sin x|
Therefore,
or
sin x cos y = ln |c sin x|
Example 6.
Solve the equation
r + sin θ − cos θ dr + r sin θ + cos θ dθ = 0
Solution:
∂M ∂N
= cos θ + sin θ = cos θ + sin θ
∂θ ∂r
Hence, the equation is exact and its solution is F = c, where
∂F ∂F
= r sin θ + cos θ and = r + sin θ − cos θ
∂θ ∂r
Solving for F using
∂F
=N
∂θ
∂F = r sin θ + cos θ ∂θ
F = r − cos θ + sin θ + T (r)
∂F
Solving for T (r) obtain ,
∂r
∂F
= − cos θ + sin θ + T 0 (r)
∂r
∂F
Since = M , then
∂r
∂F
Since = M , then
∂x
y cos (xy) + T 0 (x) = 2x + y cos (xy)
T 0 (x) = 2x
T (x) = x2
Therefore,
F = sin (xy) + x2 = c
and the set of solutions of the given equation is defined by
sin (xy) + x2 = c
Example 8.
Solve the equation
2 2
2x 3x + y − ye−x dx + x2 + 3y 2 + e−x dy = 0
Solution:
∂M 2 ∂N 2
= 2x − 2xe−x = 2x − 2xe−x
∂y ∂x
∂M ∂N
Since = , therefore the equation is exact. Hence, its solution is F = c, where
∂y ∂x
∂F 2
= M = 2x 3x + y − ye−x
∂x
∂F 2
= N = x2 + 3y 2 + e−x
∂y
Solving for F using
∂F
=N
∂y
then
∂F 2
= x2 + 3y 2 + e−x
∂y
2
∂F = (x2 + 3y 2 + e−x )∂y
2
F = x2 y + y 3 + yex + T (x)
Solving for T (x) get the derivative of F with respect to x we have,
∂F 2
= 2xy − 2xye−x + T 0 (x)
∂x
∂F
Since = M then,
∂x
2 2
2xy − 2xye−x + T 0 (x) = 2x 3x + y − ye−x
2 2
2xy − 2xye−x + T 0 (x) = 6x2 + 2xy − 2xye−x
T 0 (x) = 6x2
T (x) = 2x3
Therefore,
2
F = x2 y + y 3 + yex + 2x3
and the set of solutions of the given equation is defined by
2
x2 y + y 3 + yex + 2x3 = c
Example 9.
Solve the equation
3y(x2 − 1)dx + (x3 + 8y − 3x)dy = 0
when x = 0 and y = 1.
Solution:
∂M ∂N
= 3(x2 − 1) = 3(x2 − 1)
∂y ∂x
∂M ∂N
Since = , therefore the equation is exact. Hence, its solution is F = c, where
∂y ∂x
∂F ∂F
= M = 3y(x2 − 1) = N = x3 + 8y − 3x
∂x ∂y
Solving for F using
∂F
=M
∂x
then
∂F
= 3y(x2 − 1)
∂x
∂F = 3y(x2 − 1)∂x
3
x
F = 3y − x + T (y)
3
Solving for T (x), find the derivative of F with respect of y
3
∂F x
=3 − x + T 0 (y)
∂y 3
∂F
Since = N , then
∂y
3
x
3 − x + T 0 (y) = x3 + 8y − 3x
3
T 0 (y) = 8y
T (y) = 4y 2
Therefore,
x3
F = 3y − x + 4y 2
3
and the set of solutions of the given equation is defined by
3
x
3y − x + 4y 2 = c
3
x3 y − 3xy + 4y 2 = c
When x = 0 and y = 1
x3 y − 3xy + 4y 2 = c
c=4
x3 y − 3xy + 4y 2 = 4
x(x2 − 3) = 4(1 − y 2 )
Linear Equation of Order One
Standard Form
By dividing both sides of the linear equation by a1 (x), we obtain a more useful form,
the standard form, of a linear equation,
dx
+ P (x)y = Q(x)
dy
Suppose a linear equation in the standard form, there exist an integrating factor v(x) >
0, a function of x alone. Then the equation
dy
v(x) + P (x)y = v(x)Q(x)
dx
must be an exact equation. The equation can be written in the form
M (x, y) dx + N (x, y) dy = 0
with
M (x, y) = v(x)P (x)y − v(x)Q(x)
in which v(x), P (x) and Q(x) are functions of x alone.
P (x)dx dy
R R R
P (x)dx P (x)dx
e + P (x)ye = Q(x)e
dx
The left member of the equation is actually the derivative of the function
R
P (x)dx
ye
P (x)dx dy
R R R
P (x)dx P (x)dx
e + P (x)ye = Q(x)e
dx
is exact.
From the previous discussion, after determining the integrating factor v(x), we can
now find the solution of a first-order linear equation
dy
+ P (x)y = Q(x)
dx
and from
P (x)dx dy
R R R
P (x)dx P (x)dx
e + P (x)ye = Q(x)e
dx
From which, we have the general solution
R
Z R
P (x)dx
ye = Q(x)e P (x)dx dx + c
Example 1.
y − 1 + x cot x = c csc x
y + x cot x + c csc x = 1
Example 4.
y sin x = x + c
Example 5.
It has been found out that integrating factor is an aid in finding a solution of linear
equations. At present we are concerned enough to enable us to find integrating factors by
inspection. The ability to do this depends largely upon the recognition of certain common
exact differentials and upon experience.
d(xy) = x dy + y dx (1)
x y dx − x dy
d = (2)
y y2
y x dy − y dx
d = (3)
x x2
x y dx − x dy
d Tan−1 = (4)
y x2 + y 2
In this lecture, some equations when terms are reduced to exact differentials becomes
a more recognizable differential equation. Hence, solving them becomes easier.
Example 1.
Solve the equation
y(2xy + 1)dx − xdy = 0
Solution:
y(2xy + 1)dx − xdy = 0
2xy 2 dx + ydx − xdy = 0
1 2 1
2
2xy dx + ydx − xdy = 0 · 2
y y
ydx − xdy
+ 2xdx = 0
y2
x
d + 2xdx = 0
y
Z Z
x
d + 2xdx = c
y
x
+ x2 = c
y
x + x2 y =cy
x(1 + xy) = cy
Example 2.
Solve the equation
y(y 3 − x)dx + x(y 3 + x)dy = 0
Solution:
y(y 3 − x)dx + x(y 3 + x)dy = 0
y 3 (ydx + xdy) − x(ydx − xdy) = 0
x ydx − xdy
ydx + xdy − =0
y y2
x ydx − xdy
d(xy) − =0
y y2
Z
x ydx − xdy
Z
d(xy) − =c
y y2
( xy )2
xy − =c
2
x2
2xy − 2 = c
y
2xy − x2 = cy 2
3
Example 3.
Solve the equation
(x3 y 3 + 1)dx + x4 y 2 dy = 0
Solution:
(x3 y 3 + 1)dx + x4 y 2 dy = 0
x3 y 2 (xdy + ydx) + dx = 0
1 3 2 1
x y (xdy + ydx) + dx = 0 ·
x x
dx
x2 y 2 (xdy + ydx) + =0
Z Z x
dx
(xy)2 (xdy + ydx) + =c
x
(xy)3
+ ln |x| = ln |c|
3
x3 y 3 + 3 ln |x| = 3 ln |c|
x3 y 3 = −3 ln |cx|
Example 4.
Solve the equation
x4 (ydx + xdy) + y 2 (xdy − ydx) = 0
Solution:
x4 (ydx + xdy) + y 2 (xdy − ydx) = 0
1 4 2 1
4
x (ydx + xdy) + y (xdy − ydx) = 0 · 4
x x
2
y xdy − ydx
(xdy + ydx) + 2 =0
x x2
y 2 y
d(xy) + d =0
x x
Z Z 2
y y
d(xy) + d =c
x x
1 y 3
xy + · =c
3 x
y3
3xy + 3 = c
x
3x y + y 3 = cx3
4
y(3x4 + y 2 ) = cx3
Example 5.
Example 7.
Example 9.
Recall from Linear Equation, the form y 0 + P (x)y = Q(x) can be transformed into an
exact equation when we multiply the equation by an integrating factor. The same basic idea
sometimes apply for a non-exact differential equation M (x, y)dx + N (x, y)dy = 0. That is,
it is sometimes possible to find an integrating factor µ(x, y) so that after multiplying, the
left-hand side of
µ(x, y)M (x, y) dx + µ(x, y)N (x, y) dy = 0 (1)
is an exact differential. In an attempt to find µ(x, y), we return to the criterion for exactness.
The equation (1) is exact if and only if
∂ ∂
µ(x, y)M (x, y) = µ(x, y)N (x, y)
∂y ∂x
By the product rule of differentiation (subscript notation means partial derivatives)
µMy + M µy = µNx + N µx
or
µx N − µy M = (My − Nx ) µ (2)
Although M , N , My and Nx are known functions of x and y, the difficulty here in determining
the unknown µ(x, y) from (2) is that we must solve a partial differential equation. Since we
are not yet prepared for that, we make simplifying assumptions.
Suppose µ is a function of only one variable, e.g., µ depends only on x. In this case
dµ
µx = and µy = 0
dx
so (2) can be written as
dµ My − Nx
= ·µ (3)
dx N
My − Nx
We still at an impasse of the quotient depends on both x and y. However,
N
My − Nx
if all obvious algebraic simplifications are made, the quotient turns out to depend
N
solely on the variable x, then (3) is a first-order ordinary differential equation. We can finally
determine µ because (3) is separable and linear. It follows that
R My −Nx
dx
µ(x) = e N
In the like manner, it follows from (2) if µ depends only on the variable y, then
dµ My − Nx
=− ·µ (4)
dy M
My − Nx
In this case, if − is a function of y alone, then we can solve (4) for µ to be
M
R My −Nx
− dy
µ(y) = e M
M (x, y) dx + N (x, y) dy = 0
My − Nx R My −Nx
dx
• If is a function of x alone, the integrating factor is e N .
N
My − Nx R My −Nx
• If − is a function of y alone, the integrating factor is e − M dy .
M
Example 1.
Solve the equation
(x2 + y 2 + 1)dx + x(x − 2y)dy = 0 (1)
Solution:
Let,
M (x, y) = x2 + y 2 + 1
My = 2y
Therefore,
−y 2 x−1 − x−1 + x + y = c
−y 2 x−1 − x−1 + x + y = c
or
−x2 + y 2 − xy + 1 = cx
Example 2.
Solve the equation
2y(x2 − y + x)dx + (x2 − 2y)dy = 0 (1)
Solution:
Let
M (x, y) = 2y(x2 − y + x)
My = 2x4 − 4y + 2x
N (x, y) = x2 − 2y
Nx = 2x
My − Nx 2x4 − 4y + 2x − 2x
=
N x2 − 2y
My − Nx 2(x2 − 2y)
=
N x2 − 2y
My − Nx
=2
N
My − Nx
= µ(x)
N
Solving for the integrating factor, R
2dx
e = e2x
Multiply the integrating factor to (1)
Simplifying,
2ye2x (x2 − y + x)dx + e2x (x2 − 2y)dy = 0 (2)
Test for exactness of Equation (2),
Therefore,
x2 ye2x − y 2 e2x + c = c
x2 ye2x − y 2 e2x = c
or
y(x2 − y) = ce−2x
Example 3.
Solve the equation
y(2x − y + 1)dx + x(3x − 4y + 3)dy = 0 (1)
Solution:
Let
M (x, y) = y(2x − y + 1)
∂M
= 2x − 2y + 1
∂y
N (x, y) = x(3x − 4y + 3)
∂N
= 6x − 4y + 3
∂x
Using the formula,
My − Nx 2x − 2y + 1 − (6x − 4y + 3)
=
M y(2x − y + 1)
My − Nx −2(2x − y + 1)
=
M y(2x − y + 1)
My − Nx 2
=−
M y
My − Nx
= µ(y)
M
Solving for the integrating factor,
− y2 dx
R
e− = e2 ln y
− y2 dx
R
e− = y2
Simplifying,
y 3 (2x − y + 1)dx + xy 2 (3x − 4y + 3)dy = 0]y 2 (2)
Test for exactness of Equation (2),
M (x, y) = y 3 (2x − y + 1)
∂M
= 6xy 2 − 4y 3 + 3y 2
∂y
N (x, y) = xy 2 (3x − 4y + 3)
∂N
= 6xy 2 − 4y 3 + 3y 2
∂x
∂M ∂N ∂F
Since = , therefore (2) is exact. Find the solution F = c, using = M,
∂y ∂x ∂x
Z Z
∂F = y 3 (2x − y + 1)∂x
F = x2 y 3 − xy 4 + xy 3 + T (y)
∂F
= 3x2 y 2 − 4xy 3 + 3xy 2 + T 0 (y)
∂y
∂F
Since = N (x, y), then
∂y
Therefore,
F = x2 y 3 − xy 4 + xy 3 + T (y)
F = x2 y 3 − xy 4 + xy 3 + c
x2 y 3 − xy 4 + xy 3 + c = c
x2 y 3 − xy 4 + xy 3 = c
or
xy 3 (x − y + 1) = c
Example 4.
Solve the equation
y(4x + y)dx − 2(x2 − y)dy = 0 (1)
Solution:
Let
M (x, y) = y(4x + y)
∂M
= 4x + 2y
∂y
N (x, y) = 2(x2 − y)
∂N
= −4x
∂x
Using the formula,
My − Nx 4x + 2y − (−4x)
=
M y(4x + y)
My − Nx 2(4x + y)
=
M y(4x + y)
My − Nx 2
=
M y
My − Nx
= µ(y)
M
Solving for the integrating factor,
2
R
e− y
dx
= e−2 ln y
2
R
e− y
dx
= y −2
Simplifying,
y −1 (4x + y)dx − 2y −2 (x2 − y)dy = 0 (2)
Test for exactness of (2),
M (x, y) = y −1 (4x + y)
∂M 4x
=− 2
∂y y
N (x, y) = −2y −2 (x2 − y)
∂N 4x
=− 2
∂x y
∂M ∂N ∂F
Since = , therefore (2) is exact. Find the solution F = c, using = M,
∂y ∂x ∂x
Z Z
∂F = y −1 (4x + y)∂x
2x2
F = + x + T (y)
y
∂F 2x
= − 2 + T 0 (y)
∂y y
∂F
Since = N (x, y), then
∂y
2x2
− + T 0 (y) = −2y −2 (x2 − y)
y2
2x2 2x2 2
− 2 + T 0 (y) = − 2 +
y y y
2
T 0 (y) =
y
T (y) = 2 ln |y|
T (y) = ln (y 2 )
Therefore,
2x2
F = + x + T (y)
y
2x2
F = + x + ln (y 2 )
y
2x2
+ x + ln (y 2 ) = c
y
2x2
+ x + ln (y 2 ) = c
y
or
2x2 + xy + y ln (y 2 ) = cy
Example 5.
Let
M (x, y) = (xy + 1)
∂M
=x
∂y
N (x, y) = x(x + 4y − 2)
∂N
= 2x + 4y − 2
∂x
Using the formula,
My − Nx x − 2x + 4y − 2
=
N x(x + 4y − 2)
My − Nx −(x + 4y − 2)
=
N x(x + 4y − 2)
My − Nx 1
=−
N x
My − Nx
= µ(x)
N
Solving for the integrating factor,
− x1 dx
R
e = e− ln |x|
− x1 dx
R
e = x−1
Simplifying,
x−1 (xy + 1)dx + (x + 4y − 2)dy = 0 (2)
Test for exactness of (2),
x + T 0 (y) = x + 4y − 2
T 0 (y) = 4y − 2
T (y) = 2y 2 − 2y
Therefore,
F = xy + ln |x| + T (y)
F = xy + ln |x| + 2y 2 − 2y
xy + ln |x| + 2y 2 − 2y = C
Example 6.
Solution:
Let
M (x, y) = 2y 2 + 3xy − 2y + 6x
∂M
= 4y + 3x − 2
∂y
N (x, y) = x(x + 2y − 1)
∂N
= 2x + 2y − 1
∂x
Using the formula,
My − Nx 4y + 3x − 2 − (2x + 2y − 1)
=
N x(x + 2y − 1)
My − Nx x + 2y − 1
=
N x(x + 2y − 1)
My − Nx 1
=
N x
My − Nx
= µ(x)
N
Solving for the integrating factor,
1
R
e x
dx
= eln |x|
1
R
dx
e x =x
F = x3 y + x2 y 2 − x2 y + T (x)
∂F
= 3x2 y + 2xy 2 − 2xy + T 0 (x)
∂x
∂F
Since = M (x, y), then
∂x
3x2 y + 2xy 2 − 2xy + T 0 (x) = x(2y 2 + 3xy − 2y + 6x)
3x2 y + 2xy 2 − 2xy + T 0 (x) = 2xy 2 + 3x2 y − 2xy + 6x2
T 0 (x) = 6x2
T (x) = 2x3
Therefore,
F = x3 y + x2 y 2 − x2 y + T (x)
F = x3 y + x2 y 2 − x2 y + 2x3
x3 y + x2 y 2 − x2 y + 2x3 = c
x2 (xy + y 2 − y + 2x) = c
Example 7.
Solution:
Let
M (x, y) = y(y + 2x − 2)
∂M
= 2y + 2x − 2
∂y
N (x, y) = −2(x + y)
∂N
= −2
∂x
Using the formula,
My − Nx 2y + 2x − 2 − (−2)
=
N −2(x + y)
My − Nx 2(x + y)
=
N −2(x + y)
My − Nx
= −1
N
My − Nx
= µ(x)
N
Solving for the integrating factor,
R
−dx
e = e−x
Simplifying,
e−x y(y + 2x − 2)dx − 2e−x (x + y)dy = 0 (2)
Test for exactness of (2),
M (x, y) = ye−x (y + 2x − 2)
∂M
= 2ye−x + 2xe−x − 2e−x
∂y
N (x, y) = −2e−x (x + y)
∂N
= 2xe−x − 2e−x + 2ye−x
∂x
∂M 0 ∂N 0 ∂F
Since = , therefore (2) is exact. Find the solution F = c, using = M,
∂y ∂x ∂x
Z Z
∂F = ye−x (y + 2x − 2)∂x
Therefore,
−y 2 e−x − 2xye−x + c = c
−y 2 e−x − 2xye−x = c
−ye−x (y + 2x) = c
y(y + 2x) = cex
Example 8.
Solve the equation
y 2 dx + (3xy + y 2 − 1)dy = 0 (1)
Solution:
Let
M (x, y) = y 2
∂M
= 2y
∂y
N (x, y) = 3xy + y 2 − 1
∂N
= 3y
∂x
Using the formula,
My − Nx 2y − 3y
=
M y2
My − Nx 1
=−
M y
My − Nx
= µ(y)
M
Solving for the integrating factor,
− y1 dx
R
e− = eln |y|
− y1 dx
R
e− =y
Multiply the integrating factor to (1)
y[y 2 dx + (3xy + y 2 − 1)dy] = 0 · y
Simplifying,
y 3 dx + y(3xy + y 2 − 1)dy = 0 (2)
Test for exactness of (2),
M (x, y) = y 3
∂M
= 3y 2
∂y
N (x, y) = y(3xy + y 2 − 1)
∂N
= 3y 2
∂x
∂M ∂N ∂F
Since = , therefore (2) is exact. Find the solution F = c, using = M,
∂y ∂x ∂x
Z Z
∂F = y 3 ∂x
F = xy 3 + T (y)
∂F
= 3xy 2 + T 0 (y)
∂y
∂F
Since = N (x, y), then
∂y
F = xy 3 + T (y)
y4 y2
F = xy 3 + −
4 2
And the set of solutions of (1) is defined by
y4 y2
xy 3 + − =c
4 2
y4 y2
xy 3 + − =c
4 2
y 2 (4xy + y 2 − 2) = 4c
y 2 (4xy + y 2 − 2) = c
Example 9.
Solution:
Let
M (x, y) = 2y(x + y + 2)
∂M
= 2x + 4y + 4
∂y
N (x, y) = y 2 − x2 − 4x − 1
∂N
= −2x − 4
∂x
Using the formula,
My − Nx 2x + 4y + 4 − (−2x − 4)
=
M 2y(x + y + 2)
My − Nx 4(x + y + 2)
=
M 2y(x + y + 2)
My − Nx 2
=
M y
My − Nx
= µ(y)
M
Solving for the integrating factor,
2
R
e− y
dx
= e−2 ln |y| = y −2
Simplifying,
2y −1 (x + y + 2)dx + y −2 (y 2 − x2 − 4x − 1)dy = (2)
Test for exactness of (2),
M (x, y) = 2y −1 (x + y + 2)
∂M
= −2xy −2 − 4y −2
∂y
N (x, y) = y −2 (y 2 − x2 − 4x − 1)
∂N
= −2xy −2 − 4y −2
∂x
∂M ∂N ∂F
Since = , therefore (2) is exact. Find the solution F = c, using = M,
∂y ∂x ∂x
Z Z
∂F = 2y −1 (x + y + 2)∂x
F = x2 y −1 + 2x + 4xy −1 + T (y)
∂F
= −x2 y 2 − 4xy −2 + T 0 (y)
∂y
∂F
Since = N (x, y), then
∂y
F = x2 y −1 + 2x + 4xy −1 + T (y)
1
F = x2 y −1 + 2x + 4xy −1 + y +
y
An equation in the form M (x, y)dx + N (x, y)dy = 0 may not yield at once (or at all)
to the methods discussed in the previous discussions. Even then the usefulness of these
methods will not be exhausted. It may be possible by some change of variables to transform
the equation into a type which we know how to solve.
The term (x + 2y) appears twice in the equation and thus it attracts attention, hence we
may put
x + 2y = u
and, because no other terms of x and ystands out, we retain any one of the two.
Another equation,
(1 + 3x sin y)dx − x2 cos y dy = 0
the presence of both sin y and its differential cos y dy, and the fact that the variable y appears
in the equation in no other manner, leads us to put
u = sin y du = cos y dy
u = Ax + By + C
wherein B 6= 0.
Example 1.
Solve
(3 tan x − 2 cos y) sec2 xdx + tan x sin ydy = 0 (1)
Solution:
Let
u = tan x
du = sec2 xdx
M (x, y) = 3u − 2 cos y
My = 2 sin y
and
N (x, y) = u sin y
Nx = sin y
Then,
u3 − u2 cos y = c (3)
Since u = tan x, then (3) becomes
u = sin y
du = cos ydy
Simplifying,
x2 v(3 + v)dx + 2x3 dv = 0 (3)
Using separation of variables, divide (3) by x3 v(3 + v)
dx 2dv
+ =0
x v(v + 3)
Z Z
dx 2dv
+ =c
x v(v + 3)
2 2
ln |x| + ln |v| − ln |v + 3| = ln |c|
3 3
3 ln |x| + 2 ln |v| − 2 ln |v + 3| = 3 ln |c|
ln (u2 ) + ln |x3 | − ln (u + 3x)2 = ln |c|
u 2 x3
=c
(u + 3x)2
x3 sin2 y
=c
(3x + sin y)2
x3 sin4 y = c(3x + sin y)2
Example 4.
Solve
dy
= (9x + 4y + 1)2 (1)
dx
Solution:
We can write (1) as
dy = (9x + 4y + 1)2 dx (2)
Let
u = 9x + 4y + 1
du = 9dx + 4dy
1
dx = (du − 4dy)
9
Substituting these to (2), we have
1
dy = u2 (du − 4dy)
9
9dy = u2 (du − 4dy)
(9 + 4u2 )dy = u2 du
Separating the variables, gives
1 2 2 1
(9 + 4u )dy = u du ·
4u2 + 9 4u2 + 9
u2 du
dy = 2
4u + 9
u2 du
Z Z
dy =
4u2 + 9
1 9 2 −1 2u
y = u− Tan +c
4 16 3 3
We have,
1 3 2u
y = u − Tan−1 +c (3)
4 8 3
Since u = 9x + 4y + 1, then (3) becomes
1 3 −1 2(9x + 4y + 1)
y = (9x + 4y + 1) − Tan +c
4 8 3
−1 2(9x + 4y + 1) 18x + 2
Tan = +c
3 3
2
(9x + 4y + 1) = tan (6x + c)
3
2(9x + 4y + 1) = 3 tan (6x + c)
Example 5.
dy
= 1 + 6xex−y (1)
dx
Solution:
Let
u=x−y
du = dx − dy
dy
= 1 + 6xeu
dx
dx − du
= 1 + 6xeu
dx
dx − du = (1 + 6xeu )dx
[1 − (1 + 6xeu )]dx = du
We have,
−6xeu dx = du (2)
Diving (2) by eu to separate the variables becomes
du
−6xdx = u
Z Ze
−6xdx = e−u du
−3x2 = −e−u + c
−3x2 = −e−(x−y) + c
−3x2 = −ey−x + c
3x2 + c = ey−x
Example 6.
Solve
(x + 2y − 1)dx − (x + 2y − 5)dy = 0 (1)
Solution:
Let
u = x + 2y
du = dx + 2dy
dx = du − 2dy
We now have,
u − 4 ln |u + 3| − y = C (2)
Since u = x + 2y, then substituting to (2) we have
Solve,
(x + 2y − 1)dx + [2(x + 2y) − 3]dy = 0 (1)
Solution:
Let
u = x + 2y
du = dx + 2dy
dx = du − 2dy
We now have,
1 2
(u) − u − y = c (2)
2
Since u = x + 2y, then (2) becomes
1
(x + 2y)2 − (x + 2y) − y = c
2
(x + 2y)2 − 2(x + 2y) − 2y = c
(x + 2y)2 − 2x − 6y = c
(x + 2y − 1)2 = 2y + c
Example 8.
Solve,
y(x tan x + ln |y|)dx + tan xdy = 0 (1)
Solution:
We can rewrite (1) as
dy
(x tan x + ln |y|)dx + tan x =0 (2)
y
Let
u = ln |y|
dy
du =
y
M (x, u) = x tan x + u
∂M
=1
∂u
and
N (x, u) = tan x
∂N
= sec2 x
∂x
Using the formula,
Mu − Nx 1 − secx
=
N tan x
Mu − Nx − tan2 x
=
N tan x
Mu − Nx
= − tan x
N
Integrating,
−x cos x + sin x + u sin x = c (4)
Since u = ln |y|, then (4) gives us the result as
or
ln |y| = x cot x + c csc x − 1
Example 9.
Solve
4(3x + y − 2)dx − (3x + y)dy = 0 (1)
Solution:
Let
u = 3x + y
y = u − 3x
dy = du − 3dx
The equation
dy
+ P (x)y = Q(x)y n (1)
dx
may be put in the form
dy
y −n
+ P (x)y 1−n = Q(x) (2)
dx
Now considering y 1−n and its derivative
d dy
y 1−n = (1 − n) y −n
dx dx
Now, let
dz dy
z = y 1−n and = (1 − n) y −n
dx dx
Substituting this in (2), we will have
dz
+ (1 − n) P (x)z = (1 − n) Q(x)
dx
which is a linear equation.
The solution is
R
Z R
(1−n)P (x)dx (1−n)P (x)dx
ze = (1 − n) Q(x)e dx + c
In terms of x and y,
R
Z R
1−n (1−n)P (x)dx (1−n)P (x)dx
y e = (1 − n) Q(x)e dx + c
y −2 x3 = −x + c
x3 = (c − x)y 2
Alternate Solution
2x3 y 0 = y(y 2 + 3x2 )
y(y 2 + 3x2 )
y0 =
2x3
3
0 y 3x2
y = 3 + 3y
2x 2x
0 3 1 3
y − y = 3 ·y
2x 2x
3 dx
y −3 dy − y −2 dx = 3 (1)
2x 2x
Let
z = −y 2
dz = 2y −3 dy
x3 z = x + c (3)
1
Since z = − 2 , then (3)
y
3 1
x − 2 =x+c
y
x3 = −xy 2 + cy 2
x3 = y 2 (c − x)
Example 2.
Solution:
1 ln |y|
n=3 P (y) = − Q(y) = −
2y 2y
We have the reduced polynomials,
1 1 ln |y| ln |y|
xr = x1−3 = x−2 Pr (y) = −2 · − = Qr (y) = −2 · − =
2y y 2y y
The integrating factor is
1
R
dy
e y = eln |y| = y
The general solution is,
Z
−2 ln |y|
x y= ydy + c
y
Z
−2
x y= (ln |y|) dy + c
x−2 y = y ln |y| − y + c
y = x2 y ln |y| − x2 y + cx2
Example 3.
Solution:
y 0 = y − xy 3 e−2x
y 0 − y = −xe−2x y 3
y −2 e2x = x2 + c
e2x = y 2 x2 + c
Example 4.
Solution:
1 2 3 1
6y dx − x(2x + y)dy = 0 ·
6y 2 dy 6y 2 dy
dx 1 1
− x = 2 x4
dy 6y 3y
dx
This is a Bernoulli D.E. of the form + P (y)x = Q(y)xn with
dy
1 1
n=4 P (y) = − Q(y) =
6y 3y 2
The reduced polynomials are
1 1
xr = x−3 Pr (y) = Qr (y) = −
2y y2
The integrating factor is,
1
R 1 1
dy
e 2y = e 2 ln |y| = y 2
The general solution is
Z
−3 1 1 1
x y = 2 − 2 y 2 dy + c
y
Z
1 3
x−3 y 2 = −y − 2 dy + c
1 1
x−3 y 2 = 2y − 2 + c
1
y = x3 2 + cy 2
Example 5.
x2 y 2 = 2y 3 + c
Example 6.
(y 4 − 2xy)dx + 3x2 dy = 0
1 4 2 1
(y − 2xy)dx + 3x dy = 0 ·
3x2 dx 3x2 dx
dy 2 1
− y = − 2 y4
dx 3x 3x
This is a Bernoulli D.E. with
2 1
n=4 P (x) = − Q(x) = −
3x 3x2
The reduced polynomials are
2 1
yr = y −3 Pr (x) = Qr (x) =
x x2
The integrating factor is
2
R
e x
dx
= e2 ln |x| = x2
The general solution is
Z
−3 2 1
y x = x2 dx + c
x2
Z
−3 2
y x = dx + c
y −3 x2 = x + c
x2 = y 3 (x + c)
Example 7.
Solution:
2xyy 0 = y 2 − 2x3
1 0 2 3 1
2xyy = y − 2x
2xy 2xy
y 1
y0 = − x2
2x y
dy y
− = −x2 y −1
dx 2x
This is a Bernoulli D.E. with
1
n = −1 P (x) = − Q(x) = −x2
2x
The reduced polynomials are
1
yr = y 2 Pr (x) = − Qr (x) = −2x2
x
The integrating factor is
R
− x1 dx 1
e = e− ln |x| =
x
The general solution is
y2
Z
2
1
= −2x dx + c
x x
y2
Z
= −2x dx + c
x
y2
= −x2 + c
x
y2 = (c − x2 )x
Alternate Solution
2xyy 0 = y 2 − 2x3
1 0 2 3 1
2xyy = y − 2x
2xy 2xy
y 1
y0 = − x2
2x y
dy y 1
− = − x2
dx 2x y
dy y 1 2
y − = − x y
dx 2x y
We have,
dy 1
y − = −x2 (1)
dx 2x
Let
z = −y 2
dz = −2ydy
zx−1 = x2 + c
y2
− = x2 + c
x
y 2 = x(c − x2 )
Example 8.
Now,
dy 2y x2
y2 + = (1)
dx 3x 3
Let
z = y3
dz = 3y 2 dy
Substitute to (1) becomes
1 dy 2 x2
+ z=
3 dx 3x 3
dy 2
+ z = x2 (2)
dx x
Solving the integrating factor we have,
2dx
R
e x = e2 ln x = x2
Substituting the integrating factor to (2) becomes
dz
x2 + 2xz = x4
dx
x2 dz + 2xzdx = x4 dx
d(x2 z) = x4 dx
Z Z
d(x z) = x4 dx
2
x5
x2 z = +c
5
x5
x2 y 3 = +c
5
5x2 y 3 = x5 + C
Example 9.
y2
Z
x 1
= dx + c
x3 2 x3
y2
Z
1
= dx
x3 2x2
y2 1
=− +c
x3 2x
2y 2 = cx3 − x2
Alternate Solution
(x2 + 6y 2 )dx − 4xydy = 0
1 2 2 1
(x + 6y )dx − 4xydy = 0 ·
−4xydx −4xydx
dy x 3y
− − =0
dx 4y 2x
dy 3y x
y − = y
dx 2x 4y
We have,
dy 3 x
y − y2 = (1)
dx 2x 4
Let
z = −y 2
dz = −2ydy
Substituting to (1) becomes
1 dz 3 x
− + z=
2 dx 2x 4
Now,
dz 3 1
− z=− x (2)
dx x 2
Solving for integrating factor we have
− x3 dx
R
e = e3 ln x = x−3
Substituting the integrating factor to (2), gives us
1
x−3 dz − 3x−4 dx = − x−2 dx
2
1
d(x3 z) = − x−2 dx
Z Z2
1
d(x−3 z) = − x−2 dx
2
1 1
x−3 z = · + c
2 x
y2 1 1
− 3 = · +c
x 2 x
y2 1
− 3 = +c
x 2x
2y 2 = −x2 + cx3
Equations with Coefficients Linear in the Two Variables
a1 x + b1 y + c1 = 0 a2 x + b 2 y + c 2 = 0 (2)
These lines may be parallel or they may intersect. There will not be two lines if a1 = b1 = 0
or a2 = b2 = 0, but equation (1) will then be linear in one of its variables.
will change the equations (2) into equation of line through the origin of the uv -coordinate
system, namely,
a1 u + b 1 v + c 1 = 0 a2 u + b 2 v = 0 (4)
Therefore, since dx = du and dy = dv, the change of variables
x=u+h y =v+k
where (h, k) is the point of intersection of the lines in (2), will transform (1) into
a2 x + b2 y = k(a1 x + b2 y)
Solution:
The lines
y−2=0 and x−y−1=0
intersect at (3, 2). Let
x=u+3
dx = du
and
y =v+2
dy = dv
v = ru
dv = rdu + udr
Alternate Solution
Equation (2) can be written as
vdu − (u − v)dv = 0
du u
− = −1
dv v
This is linear in u, with
1
P (v) = − Q(v) = −1.
v
The integrating factor is
R
− v1 dv −1 1
e = e− ln v = ev = .
v
The general solution is
Z
u 1
= −1 dv + c
v v
u
= − ln |v| − ln |c|
v
u = −v ln |cv|
Since u = x − 3 and v = y − 2, then
x − 3 = (2 − y) ln |c(y − 2)|
Example 2.
Solve the equation
(x − 4y − 9)dx + (4x + y − 2)dy = 0 (1)
Solution:
The lines
x − 4y − 9 = 0 and 4x + y − 2 = 0
intersect at (1, −2). Let
x=u+1 dx = du
and
y =v−2 dy = dv
Then (1) becomes
(u − 4v)du + (4u + v)dv = 0 (2)
Let
v = ru
dv = rdu + udr
substitute v and dv to transform (2) into
(u − 4ru)du + (4u + ru)(rdu + udr) = 0
u(1 + r2 )du + u2 (4 + r)dr = 0
du r+4
+ 2 dr = 0
Z u Z r +1
du r+4
+ dr = c
u r2 + 1
1
ln |u| + ln |r2 + 1| + 4 Tan−1 r = c
2
ln |u2 (r2 + 1)| + 8 Tan−1 r = c
v
Since r = , then
u
2 v 2 + u2
−1 v
ln u
2
+ 8 Tan =c
u u
and since u = x − 1, v = y + 2 then
2
2 v + u2
ln u + 8 Tan−1 v = 0
u2 u
x−1
ln |(y + 2)2 + (x − 1)2 | + 8 Tan−1 =0
y+2
Example 3.
Solution:
The lines
2x − y = 0 and 4x + y − 6 = 0
lines intersect at (1, 2). Let
x=u+1 dx = du
and
y =v+2 dy = dv
Then (1) becomes
(2u − v)du + (4u + v)dv = 0 (2)
Let
v = ru
dv = rdu + udr
(v + u)3
=c
(v + 2u)2
3
(y − 2) + (x − 1)
2 =c
(y − 2) + 2(x − 1)
(x + y − 3)3
=c
(2x + y − 4)2
(x + y − 3)3 = c(2x + y − 4)2
Example 4.
Solution:
The lines
x − 4y − 3 = 0 and x − 6y − 5 = 0
intersect at (−1, −1). Let
x=u−1 dx = du
and
y =v−1 dy = dv
Then (1) becomes
(u − 4v)du − (u − 6v)dv = 0 (2)
Let
v = ru
dv = rdu + udr
Solution:
The lines
2x + 3y − 5 = 0 and 3x − y − 2 = 0
intersect at (1, 1). Let
x=u+1 dx = du
and
y =v+1 dy = dv
Then (1) becomes
(2u + 3v)du + (3u − v)dv = 0 (2)
Let
v = ru
dv = rdu + udr
substitute v and dv to transform (2) into
(2u + 3ru)du + (3u − ru)(rdu + udr) = 0
(2 + 6r − r2 )udu + u2 (3 − r)dr = 0
du 3−r
+ dr = 0
u Z 2 + 6r − r2
r−3
Z
du
+ dr = c
u 2 + 6r − r2
1
ln |u| + ln |2 + 6r − r2 | = ln |c|
2
ln |u2 (2 + 6r − r2 )| = ln |c|
u2 (2 + 6r − r2 ) = c
v
Since r = , then
u
2
2 v v
u 2+6 − =c
u u
2u2 + 6uv − v 2 = c
and since u = x − 1, v = y − 1 then
2(x − 1)2 + 6(x − 1)(y − 1) − (y − 1)2 = c
2x2 − y 2 + 6xy − 10x − 4y = c
Example 6.
Solution:
Let
u=x+y
du = dx + dy
Since u = x + y, then
x − 2u + 3 ln |u + 2| = c
x − 2(x + y) + 3 ln |(x + y) + 2| = c
x + 2y + c = 3 ln |x + y + 2|
Example 7.
Solve the equation
(x − 2)dx + 4(x + y − 1)dy = 0 (1)
Solution:
The lines
x−2=0 and 4(x + y − 1) = 0
intersect at (2, −1). Let
x=u+2 dx = du
and
y =v−1 dy = dv
Then (1) becomes
udu + 4(u + v)dv = 0 (2)
Let
u = rv
du = rdv + vdr
substitute v and dv to transform (2) into
rv(rdv + vdr) + 4(rv + v)dv =0
v(r2 + 4r + 4)dv + rv 2 dr =0
dv rdr
+ =0
v (r + 2)2
Z Z
dv rdr
+ =c
v (r + 2)2
2
ln |v| + ln |r + 2| + = − ln |c|
r+2
2
= − ln cv(r + 2)
r+2
u
Since r = , then
v
2 u
u = − ln cv
+ 2
+2 v
v
2v
= − ln c(u + 2v)
u + 2v
and since u = x − 2, v = y + 1 then
2(y + 1)
= − ln |c[(x − 2) + 2(y + 1)]|
(x − 2) + 2(y + 1)
2(y + 1) = −(x + 2y) ln |c(x + 2y)|
Example 8.
Solve the equation
(x − 1)dx − (3x − 2y − 5)dy = 0 (1)
Solution:
The lines
x−1=0 and 3x − 2y − 5 = 0
intersect at (1, −1). Let
x=u+1 dx = du
and
y =v−1 dy = dv
Then (1) becomes
udu − (3u − 2v)dv = 0 (2)
Let
u = rv du = rdv + vdr
substitute v and dv to transform (2) into
rv(rdv + vdr) − (3rv − 2v)dv = 0
rv 2 dr + (r2 v − 3rv + 2v)dv = 0
rdr dv
2
+ =0
Z r − 3r + 2 Z v
rdr dv
2
+ =c
r − 3r + 2 v
2 ln |r − 2| − ln |r − 1| + ln |v| = ln |c|
v(r − 2)2
ln = ln |c|
r−1
v(r − 2)2
=c
r−1
v(r − 2)2 = c(r − 1)
u
Since r = , then
v
u 2 u
v −2 =c −1
v v
2
(u − 2v) = c(u − v)
and since u = x − 1, v = y + 1 then
2
(x − 1) − 2(y + 1) = c [(x − 1) − (y + 1)]
(x − 2y − 3)2 = c(x − y − 2)
Example 9.
Solve the equation
(2x − 3y + 4)dx + 3(x − 1)dy = 0 (1)
Solution:
The lines 2x − 3y + 4 = 0 and 3(x − 1) = 0 intersect at (1, 2). Let
x=u+1 dx = du
and
y =v+2 dy = dv
Then (1) becomes
(2u − 3v)du + 3udv = 0 (2)
Let
v = ru du = rdv + vdr
substitute v and dv to transform (2) into
v
Since r = , then
u
2 ln |u| + 3r = c
v
2 ln |u| + 3 =c
u
and since u = x − 1, v = y − 2 then
v
2 ln |u| + 3 =c
u
y−2
2 ln |x − 1| + 3 =c
x−1
Bibliography
[Boyce and Diprima, 2001] Boyce, W. and Diprima, R. (2001). Elementary Differential
Equations with Boundary Value Problems. John Wylie & Sons, Inc.
[Bronson and Costa, 2009] Bronson, R. and Costa, G. (2009). Schaum’s Outline of
Differential Equations. McGraw-Hill Education, 3rd edition.
[Lebl, 2019] Lebl, J. (2019). Notes on Diffy Qs: Differential Equations for Engineers.
Independently Published.
[Rainville et al., 2013] Rainville, D., Bedient, P., and Bedient, R. (2013). Elementary
Differential Equations: Pearson New International Edition. Pearson Education, Limited.
[Zill, 2012] Zill, D. (2012). A First Course in Differential Equations with Modeling
Applications. Cengage Learning.
Module 3 Modeling using First-Order
Differential Equations
Introduction
In this module, we will discuss real life applications of first-order differential equation.
Modeling is the process of writing a differential equation to describe a physical
situation/phenomena. Almost all of the differential equations that you will use in your job
as engineers are there because somebody, at some time, modeled a situation to come up
with the differential equation that you are using. This module is not intended to
completely teach you how to go about modeling all physical situations. This module is
designed to introduce you to the process of modeling and show you what is involved in
modeling.
In all of these situations we will be forced to make assumptions that do not accurately
depict reality in most cases, but without them the problems would be very difficult and
beyond the scope of this discussion
Topic Outcomes
2. Show solution of some linear models of systems using the appropriate method,
expressed in simplest form or conditions identified.
3. Show solution of some non-linear models of systems using the appropriate method,
expressed in simplest form or conditions identified
Differential Equations as Mathematical Model
Mathematical Model
It is often desirable to describe the behavior of some real life system or phenomenon,
whether physical, sociological or even economic in mathematical terms. The mathematical
description of a system or a phenomenon is called a mathematical model and is constructed
with certain goals in mind.
Note: increasing the resolution adds complexity to the mathematical model and more likely
that an explicit solution cannot be obtained
A mathematical model of a physical system will often involve the variable time t. A
solution of the model then gives the state of the system; in other words, the values of the
dependent variables for appropriate values of t describe the system in the past, present and
future.
Population Dynamics
This simple model, however, fails to take into account may factors that can influence
human population to grow or decline (e.g. immigration, emigration and death), nevertheless
turned out to be fairly accurate in predicting population of the United States during the
years 1790-1860. Populations that grow at a rate described by the Malthusian assumption
are rare; nevertheless, it is still used to model growth of small population over short intervals
of time (e.g. bacteria in a petri dish).
Radioactive Decay
The nucleus of an atom consists of protons and neutrons. Many of these combinations
of protons and neutrons are unstable – that is, the atoms decay or transmute into atoms of
another substance. Such nuclei are said to be radioactive. For example, over time the highly
radioactive radium, Ra-226, transmutes into the radioactive gas radon, Rn-222. To model
this phenomenon of radioactive decay, it is assumed that the rate of at which a radioactive
substance decay is directly proportional to the amount (more precisely, the number of nuclei)
of the substance remaining at time t is
dA dA
∝A or = kA.
dt dt
where A is the amount of the substance remaining.
dS
The model of growth can also be seen as the equation = rS, which describes
dt
the growth of capital S when an annual interest rate of r is compounded continuously. In
biological application the decay model can also be used to determine the half-life of a drug,
which is the time 50% of the drug is eliminated from the body by excretion or metabolism.
In chemistry the decay model appears as the model for the first-order chemical reaction.
The point is that a single differential equation can serve as a mathematical model for many
different phenomena.
Newton’s Law of Cooling/Warming
Spread of Disease
A contagious disease, for example a flu virus, is spread throughout the community by
people coming into contact with other people. Let us denote x(t) as the number of people
infected and y(t) as the number of people not yet exposed. It can be reason out that the rate
dx
at which the disease spreads is proportional to the number of interactions or encounters
dt
of the two groups of people. If we assume that the interaction is jointly proportional to both
groups, that is
dx dx
∝ xy or = kxy
dt dt
Suppose a small community with a fixed population of n people. If one person is introduced
into the community it can be argued that x(t) + y(t) = n + 1. Now our model becomes
dx
= kx(n + 1 − x)
dt
An obvious initial condition here is that x(0) = 1.
Chemical Reactions
Mixtures
The mixing of two salt solutions of differing concentrations gives rise to a first-order
differential equation for the amount of salt contained in the mixture. Let us suppose that a
large mixing tank initially holds a solution/mixture of volume V0 (m3 ), containing an amount
S0 (kg) of dissolved substance. Another solution/mixture whose concentration is Ci (kg/m3 )
enters the tank at a rate of Ri (m3 /min) while simultaneously a well-stirred solution whose
concentration is Co (kg/m3 ) leave the tank at a rate of Ro (m3 /min). If S(t) denotes the
amount of salt in the tank at any time t, the rate at which S(t) changes is a net rate:
dS
= Ri Ci − Ro Co
dt
S
where C0 = . There are three possibilities for the input and output rate of
V0 + (Ri − Ro )t
the solutions: Ri = Ro , Ri > Ro and Ri < Ro . In the latter two cases, the volume of the
solution in the tank is either increasing (Ri > Ro ) or decreasing (Ri < Ro ) at a net rate of
Ri − Ro .
Draining a Tank
In hydrodynamics, Torricelli’s Law states that the speed v of efflux of water through
a sharp-edge hole at the bottom of a tank filled to a depth h is the same as the speed that
a bodyp (in this case a drop of water) would acquire in falling freely from a height h, i.e.
v = 2gh, where g is the acceleration due to gravity. This last expression comes from
1
equating the kinetic energy mv 2 with the potential energy mgh and solving for v. If the
2 p
area of the hole is Ah and the speed of the water p leaving the tank is v = 2gh, then the
volume of water leaving the tank per second is Ah 2gh. Thus if V (t) denotes the volume
of water in the tank at time t, then
dV p
= −Ah 2gh
dt
where the minus sign indicates that V is decreasing. Note that we are ignoring the possibility
of friction at the hole that might cause a reduction of the rate of flow there. Now, if the tank
is such that the volume of water in it a time t can be written V (t) = Aw h, where Aw is the
dV dh
constant area of the upper surface of the water, then = Aw . Hence the differential
dt dt
Ah √
equation for the height of the water at any time t dh dt
= − Aw
2gh It is interesting to note
that the previous equation is valid even when Aw is not constant. In this case we must
express the upper surface area of the water as a function of h, i.e. Aw = A(h).
Series Circuits
Falling Bodies
To construct a mathematical model of the motion of a body moving in a force field, one
starts with Newton’s second law of motion. Recall from elementary physics that Newton’s
first law of motion states that a body will either will remain at rest or will continue to move
with a constant velocity unless acted by an externalX force. In each case this is equivalent
to saying that when the sum of the forces F = Fk ,i.e the net resultant forces acting
on the body is zero, then the acceleration a of the body is zero. Newton’s second law of
motion indicates that when the net force acting on a body is not zero, then the net force is
proportional to its acceleration a or, more precisely, F = ma, where m is the mass of the
body.
Now supposed a rock is tossed upward the roof of a building. What is the position
s(t) of the rock relative to the ground at time t? The acceleration of the rock is the second
ds
derivative 2 . If we assume that the upward direction is positive and that no force acts on
dt
the rock other that the force of gravity, then Newton’s second law gives
d2 s d2 s
m = −mg or = −g
dt2 dt2
In other words, the net force is simply the weight F = −W of the rock near the surface
of the earth. Recall that the magnitude of the weight is W = mg, where m is the mass of the
body and g is the acceleration due to gravity. The minus sign indicates that the direction is
downwards. If the height of the building is s0 and the initial velocity of the rock v0 , then s
is determined from the second-order initial value problem
d2 s
= −g subject to s(0) = s0 s0 (0) = v0
dt2
Although we have not been stressing solutions of the equations we have constructed, note
that the equation can be solved by integrating the constant −g twice with respect to t. the
initial conditions determine the two constant of integration. From elementary physics you
1
might recognize the solution as s(t) = − gt2 + v0 t + s0 .
2
Falling Bodies and Air Resistance
Before Galileo’s famous experiment from the leaning tower of Pisa, it was generally
believed that heavier objects in free fall, such as a cannonball and a feather when dropped
simultaneously from the same height do fall at different rates, but it is not because a
cannonball is heavier. The difference in rates is due to air resistance. The resistive force of
air was ignored in the model previously given. Under some circumstances a falling body of
mass m, such as a feather with low density and irregular shape, encounters air resistance
proportional to its instantaneous velocity v. if we take, in this circumstance, the positive
direction to be oriented downward, then the net force acting on the mass is given by
F = F1 + F2 = mg − kv, where where F1 = mg of the body is force acting on the positive
direction and air resistance F2 = −kv is a force called viscous damping, acting in the
dv
opposite or upward direction. Now, since v is related to acceleration a by a = , Newton’s
dt
dv
second law becomes F = ma = m . By equating the net force to this of Newton’s second
dt
law, we obtain a first-order differential equation for the velocity v(t) of the body at time t,
dv
m = mg − kv
dt
Here k is a positive constant of proportionality. If s(t) is the distance the body falls in time
ds dv d2 s
t from its initial point of release, then v = and a = = 2 . In terms of s, the equation
dt dt dt
is a second-order differential equation
d2 s ds d2 s ds
m 2
= mg − k or m 2
+ k = mg.
dt dt dt dt
Remarks
Each example discussed in this lecture described a dynamical system- a systems that
changes or evolves with the flow of time. Since the study of dynamical systems is a branch
of mathematics currently in vogue, we shall occasionally relate the terminology of the field
discussion in hand.
In more precise terms, a dynamical system consists of a set of time dependent variables,
called state variables, together with a rule that enables us to determine (without ambiguity)
that state of the systems (this may be a past, present of future state) in terms of a state
prescribed at some time t0 . Dynamical systems are classified as either discrete-time systems
or continuous-time systems. In this course we shall be concerned only with continuous-time
systems-systems in which all variables are defined over a continuous range of time. The rule,
or mathematical model, in a continuous-time dynamical system is a differential equation
or a system of differential equations. The state of the system at time t is the value of the
state variables at that time; the specified state of the system at a time t0 is simply the
initial conditions that accompany the mathematical model. The solution of the initial-value
problem is referred to as the response of the system.
Note that not every system studied in this course is a dynamical system. We shall also
examine some static systems in which the model is also a differential equation.
Linear Models
Suppose a culture has P0 number of bacteria. After 1 hour, the number of bacteria
doubled. If the rate of growth is proportional to the number of bacteria P (t) present at time
t, determine the time that the population triples.
Solution:
Let P be the bacterial population, we have
dP
= kP
dt
Solving the differential equation we have,
P = cekt
Now at t = 0, P (0) = P0 ,
P0 = ce(k)(0)
Hence,
c = P0
Now, the model becomes,
P = P0 ekt
At t = 1, P = 2P0
2P0 = P0 e(k)(1)
Now, for ek
ek = 2
The, the models becomes
P = P0 (2t )
Now, for t when P = 3P0
3P0 = P0 (2t )
Now,
t = 1.58 hr
Solution:
Let A(t) be the remaining amount,
dA
= kA A(0) = A0
dt
Solving for A,
A = cekt
For c, A = A0 ek·0 ,
c = A0
Hence,
A = A0 ekt
At t = 15 years, A = (1 − 0.043%)A0 = 0.99957A0 . Now, for ek ,
0.99957A0 = A0 ek·15
1
ek = 0.99957 15
Now, the equation becomes
t
A = A0 0.99957 15
To solve for t when A = 0.5A0 ,
t
0.5A0 = A0 0.99957 15
A fossilized bone is found to contain one-thousandth of the C-14 level found in living
matter. Estimate the age of the fossil. (Half-life of C-14 is 5, 600 years)
Solution:
Carbon dating also uses the model
dA
= kA
dt
Hence,
A = A0 ekt
as solved in the earlier examples. For ek , at t = 5, 600 years, A = 0.5A0
0.5A0 = A0 ek(5600)
Hence,
1
ek = 0.5 5600
Now, the equation becomes
t
A = A0 0.5 5600
A0
For t at A =
1000
A0 t
= A0 0.5 5600
1000
t = 55, 808 yrs
When a cake is removed from an oven, its temperature is measured at 300◦ F. Three
minutes later its temperature is 200◦ F. How long will it take for the cake to cool off to 80◦
F if the room temperature is 70◦ F?
Solution:
The model is
dT
= k(t − 70) T (0) = 300
dt
Rearranging the model we have,
dT
= k dt
T − 70
Solving the differential equation,
T = cekt + 70
To solve for c impose the initial condition,
300 = cek·0 + 70
Hence,
c = 230
Now, T becomes,
T = 230ekt + 70
For ek , T = 200 at t = 3 min
200 = 230ek·3 + 70
13
k 13
e =
23
Then for T ,
3t
13
T = 230 + 70
23
Solving for t when T = 80
t
13 3
80 = 230 + 70
23
t = 16.49 min.
Mixtures
The mixture of two fluids sometimes gives rise to a linear first-order differential
equation. As discussed in the previous discussions the rate at which the mixing of salt is
dS
= Ri Ci − Ro Co
dt
where,
S
Co =
V0 + (Ri − Ro )t
Example 5. Mixture of Two Salt Solutions
Suppose a large mixing tank initially holds 300 gallons of water in which 100 pounds
of salt have been dissolved. Another brine solution is pumped into the tank at a rate of 3
gal/min, and when the solution is well-stirred, it is then pumped out at a rate of 3 gal/min.
If the concentration of the solution entering is 2 lb/gal, determine the amount of salt inside
the tank after 30 minutes of pumping.
Solution:
The model is
dS
= Ri Ci − Ro Co
dt
dS Ro
+ S = Ri Ci
dt V0 + (Ri − Ro )t
wherein,
S(0) = 100 lb
Now, the working model becomes
dS 3
+ S =3·2
dt 300 + (3 − 3)t
or
dS S
+ = 6.
dt 100
This is a linear differential equation with
1
P (t) = Q(t) = 6.
100
The integrating factor is
1 t
R
dt
e 100 = e 100
The general solution is
Z
t t
Se 100 = 6e 100 + c
t t
Se 100 = 600e 100 + c
Isolating S,
t
S = 600 + ce− 100
Now, let us impose the initial condition,
0
100 = 600 + ce− 100
c = −500
Series Circuits
First-order series circuits are circuits that contain only the passive element resistor and
one and only one type of reactive element either the inductor or the capacitor. For a series
R-L circuit with resistance R and inductance L and the impressed voltage E(t), the equation
for the current i(t) is
di
L + Ri = E(t)
dt
The current i(t) here is the response of the system. For a series R-C circuit, the equation is
q
+ Ri = E(t)
C
dq
since i = , we can have
dt
dq q
R + = E(t)
dt C
Both of these equations are linear.
Example 6. Series R-L Circuit
A 12-volt battery is connected to a series R-L circuit in which the inductance is 0.5
H and the resistance is 10 Ω. Determine the current i at any time t if the initial current is
zero.
Solution:
Working equation,
di
L + Ri = E
dt
Substitute the values,
di
0.5 + 10i = 12
dt
di
+ 20i = 24
dt
The equation is linear with
P (t) = 20 Q(t) = 24
ie20t = 1.2e20t + c
i = 1.2 + ce−20t
0 = 1.2 + ce−20·0
c = −1.2
Population Dynamics
Let us say that P (t) represents the size of a population at time t, the model for
dP
exponential growth begins with the assumption that = kP for some k > 0. In this
dt
model, the relative or specific growth rate is defined by
dP
k = dt . (1)
P
True cases of exponential growth over long periods of time are hard to find because the
limited resources of the environment will at some time exert restriction on the growth of the
population. Thus, for other models, (1) can be expected to decrease as the population P
increases in size.
The assumption that the population grows (or declines) is dependent only on the
number present and not on any time-dependent mechanism such a seasonal phenomenon
can be stated as
dP
dt = f (P ) dP
or = P f (P ) (2)
P dt
The differential equation in (2) is widely assumed in models of animal population and is
called the density-dependent hypothesis.
Logistic Equation
dP
= P (a − bP ) (4)
dt
Equation (4) is known to be the logistic equation, and its solution is called the logistic
function. The graph of this logistic function is called logistic curve.
One method of solving (4) is separation of variables. Decomposing (4) into partial
fractions we have
1 b
a a
+ dP = dt
P a − bP
Solving we have,
ac1 eat ac1
P (t) = =
1 + bc1 e at bc1 + e−at
a P0
6
IF P (0) = 0, P0 = , we find c1 = , substituting this the solution becomes
b a − bP0
aP0
P (t) = (5)
bP0 + (a − bP0 ) e−at
Solution:
Assuming that no one leaves the campus during the duration of the disease, we must solve
the initial-value problem
dx
= kx(1000 − x) x(0) = 1
dt
By solving the equation we will arrive at
1000k 1000
x(t) = −1000kt
=
k + 999ke 1 + 999e−1000kt
Now, since x(4) = 50
1000
50 =
1 + 999e−1000k(4)
1
19 4
e−1000k =
999
Now, the solution becomes
1000
x(t) = t
19 4
1 + 999
999
Finally,
1000
x(6) = 6
19 4
1 + 999
999
x(6) = 276 students
There are many variations of the logistic equation. For example, the differential
equations
dP dP
= P (a − bP ) − h and = P (a − bP ) + h (6)
dt dt
could serve, in turn, as models for the population in a fishery where fish are harvested or
are restocked at a rate h. When h > 0 is a constant (6) can be easily solve by separating
variables.
Another equation in the form (2)
dP
= P (a − b ln P ) (7)
dt
is a modification of the logistic equation known as the Gompertz differential equation. This
DE can be used as a model for population growth or decay, growth of solid tumors and
certain kinds of actuarial predictions.
Bibliography
[Boyce and Diprima, 2001] Boyce, W. and Diprima, R. (2001). Elementary Differential
Equations with Boundary Value Problems. John Wylie & Sons, Inc.
[Bronson and Costa, 2009] Bronson, R. and Costa, G. (2009). Schaum’s Outline of
Differential Equations. McGraw-Hill Education, 3rd edition.
[Lebl, 2019] Lebl, J. (2019). Notes on Diffy Qs: Differential Equations for Engineers.
Independently Published.
[Rainville et al., 2013] Rainville, D., Bedient, P., and Bedient, R. (2013). Elementary
Differential Equations: Pearson New International Edition. Pearson Education, Limited.
[Zill, 2012] Zill, D. (2012). A First Course in Differential Equations with Modeling
Applications. Cengage Learning.
Module 4 Higher-Order Linear
Differential Equations
Introduction
In this module, we will discuss the definitions and forms of higher-order linear
differential equation. We will also have discussions on how to determine whether functions
a linearly independent with each other. The concept of the differential operator will be
introduced here.
Topic Outcomes
1. Recognize the general linear equation and identify its order and homogeneity by writing
a DE into the general linear form.
4. Write a DE using the differential operator form correctly and vice versa.
The general linear differential equation on order n can be written in the form
dn y dn−1 y dy
b0 (x) + b 1 (x) + · · · + b n−1 (x) + bn (x)y = R(x) (1)
dxn dxn−1 dx
If the value of R(x) is zero for all values of x, then the equation is a homogeneous differential
equation. A linear differential equation has constant coefficients if all coefficients bi (x) are
constant, if one or more of these coefficients is not constant it has variable coefficients. If the
coefficients bi (x) and the function R(x) are continuous on the interval I and b0 (x) is never
zero on I, then equation (1) is said to be normal on I.
Example 1.
Identify the order, linearity and homogeneity of the differential equation and the
interval in which the equation is normal.
(x + 1)y 0 + v = cos x
Example 2.
Identify the order, linearity and homogeneity of the differential equation and the
interval in which the equation is normal.
d2 y
3 + xy = 0
dx2
The equation is a second-order linear homogeneous differential equation and is normal on
any interval.
has a solution y1 and y2 and if c1 and c2 are constants hence by Theorem 1 equation (2) has
y = c1 y1 + c2 y2
as another solution.
Similarly, it can be seen that if yi , with i = 1, 2, · · · , k are solutions of Equation (2)
and if ci , wtih i = 1, 2, · · · , k are constants then
y = c1 y1 + c2 y2 + · · · + ck yk
is also a solution of (2).
Example 3.
Find the unique solution to the initial-value problem
y 00 + y = 0, y(0) = 0, y 0 (0) = 1
Solution:
We can observe that sin x and cos x are solutions of the differential equation, so that for
arbitrary c1 and c2
y = c1 sin x + c2 cos x (1)
is also a solution.
Differentiating (1),
y 0 = c1 cos x − c2 sin x (2)
Imposing the initial conditions in (1) and (2), we have
c1 sin 0 + c2 cos 0 = 0
c2 = 0
and
c1 cos 0 − c2 sin 0 = 1
c1 = 1
Hence, the unique solution at any interval for the problem is
y = sin x
Example 4.
Solution:
The equation presented is normal on either x > 0 or x < 0. Since the initial condition stated
is x0 = 1 we will let the interval I be the interval x > 0. We can observe that x3 and x−4
are solutions of the differential equation, so that for arbitrary c1 and c2
y = c1 x3 + c2 x−4 (3)
c1 + c2 = 4
and
3c1 − 4c2 = 5
Thus, it could be found out that c1 = 3 and c2 = 1, and therefore the function
y = 3x3 + x−4
satisfies the initial-value problem for x > 0. And by Theorem 2 the solution that we have
found out is the only solution of the initial-value problem valid for x > 0 at x = 1.
Linear Independence
Given the functions f1 , f2 , · · · , fn and if the constants c1 , c2 , · · · , cn not all zero exist
such that
c1 f1 (x) + c2 f2 (x) + · · · + cn fn (x) = 0 (1)
for all x in some interval a ≤ x ≤ b, then the functions f1 , f2 , · · · , fn are said to be linearly
dependent on that interval. If no such relations exists, the function are said to be linearly
independent. That is, the functions f1 , f2 , · · · , fn are linearly independent on an interval
when (1) implies that c1 = c2 = · · · = cn = 0 It should be clear that if a function of a set
is linearly dependent, at least one of them is a linear combination of the others; if they are
linearly independent, then none of them is a linear combination of the others.
The Wronskian
With the definition of linear independence, we shall now obtain a sufficient condition
that n functions be linearly independent on an interval a ≤ x ≤ b. Let us assume that each
of the functions f1 , f2 , · · · , fn is differentiable at least (n−1) times in the interval a ≤ x ≤ b.
Then from the equation
For any fixed value of x in the interval a ≤ x ≤ b, the nature of the solution of these
n linear equations in c1 , c2 , · · · , cn , will be determined by the value of the determinant
f1 (x)
0 f2 (x) ··· fn (x)
f1 (x) f20 (x) ··· fn0 (x)
W (x) = .. .. ..
...
. . .
(n−1) (n−1) (n−1)
f1 (x) f2 (x) · · · fn (x)
If W (x) 6= 0 for some x0 on the interval a ≤ x ≤ b, then it follows that c1 , c2 , · · · , cn ,
and hence the functions f1 , f2 , · · · , fn are linearly independent on a ≤ x ≤ b.
The function W (x) is called the Wronskian determinant, or simply Wronskian, of the
n functions f1 , f2 , · · · , fn . We have shown that if at one point on the interval the Wronskian
is not zero, then the functions are linearly independent on that interval.
Theorem 3
Example 1.
Show that ex , e2x , e3x are linearly independent.
Solution:
x 2x 3x
e e e
W (x) = ex 2e2x 3e3x
ex 4e2x 9e3x
W (x) = 18e6x + 3e6x + 4e6x − 2e6x − 12e6x − 9e6x
W (x) = 2e6x
Since W (x) = 2e2x 6= 0, then ex , e2x , e3x are linearly independent.
Example 2.
Show that ex , cos x, sin x are linearly independent.
Solution:
x
e
x cos x sin x
W (x) = e − sin x cos x
ex − cos x − sin x
W (x) = ex sin2 x + ex cos x − ex sin x cos x + ex sin2 x + ex cos2 x + ex sin x cos x
W (x) = 2ex sin2 x + 2ex cos2 x
W (x) = 2ex (sin2 x + cos2 x)
W (x) = 2e6x
Since W (x) = 2e2x 6= 0, then ex , cos x, sin x are linearly independent.
Example 3.
Show that cos (ωt − β), cos ωt, sin ωt are linearly dependent functions of t.
Solution:
cos (ωt − β) cos ωt sin ωt
W (t) = −ω sin (ωt − β) −ω sin ωt ω cos ωt
−ω 2 cos (ωt − β) −ω 2 cos ωt −ω 2 sin ωt
W (t) = 0
Since W (t) = 0, then cos (ωt − β), cos ωt, sin ωt are linearly dependent.
Example 4.
Solution:
1 sin2 x cos2
x
W (x) = 0 sin 2x − sin 2x
0 2 cos 2x −2 cos 2x
W (x) = −2 sin 2x cos 2x + 0 + 0 − 0 + 2 sin 2x cos 2x − 0
W (x) = 0
Theorem 4
If φ is any solution of (1), valied on the interval a ≤ x ≤ b, there exist constants c̄1 , c̄2 ,
· · · , c̄n such that
φ = c̄1 y1 + c̄2 y2 + · · · + c̄n yn .
y = c1 y1 + c2 y2 + · · · + cn yn
Let yp be any particular solution (not having any arbitrary constants) of the equation
Then,
y = yc + yp (4)
is a solution of (2).
yc = c1 y1 + c2 y2 + · · · + cn yn (5)
in which c1 , c2 , · · · , cn are arbitrary constants, is the general solution of (3). The right
member of equation (5) is called the complementary function for equation (2).
The general solution of a non-homogeneous linear differential equation is the sum of
the complementary function and any particular solution.
Example 1.
Solution:
We will learn later that 1 and x are linearly independent solution on any interval and are
solutions of the homogeneous equation y 00 = 0. Hence the complementary function of the
equation is
yc = c1 + c2 x
On the other hand the function 2x2 is a particular solution of the equation. Hence the
general solution is
y = yc + yp
y = c1 + c2 x + 2x2
Example 2.
Solution:
It is obvious that y = −4 is a solution. Hence we can take yp = −4. We shall see in the next
topics that the equation y 00 − y = 0 has a general solution to be
yc = c1 ex + c2 e−x
dn y dn−1 y dy
Ay = a0 n
+ a 1 n−1
+ · · · + an−1 + an y
dx dx dx
The coefficients of the operator may be constants or functions of the independent variable.
Two differential operators A and B are equal if when the same result is obtained upon
performing the operation on the same function.
A = B ⇐⇒ Ay = By
Notes:
Differential operators are linear operators; i.e, if A is any differential operator, c1 and
c2 are constants, and f1 and f2 are any function of x each possessing the required number
of derivatives, then
A(c1 f1 + c2 f2 ) = c1 Af1 + c2 Af2
Fundamental Laws of Operation
A+B =B+A
(A + B) + C = A + (B + C)
(AB)C = A(BC)
A(B + C) = AB + AC
e. If A and B are operators with constant coefficients, then they also satisfy the
commutative law of multiplication
AB = BA
Example 1.
Solution:
dy
Ay = + 2y
dx
dy
By = 3 − y
dx
Then,
dy
ABy = (D + 2) 3 − y
dx
2
dy dy
ABy = 3 2 + 5 − 2y
dx dx
2
ABy = (3D + 5D − 2)y
Hence, we have
AB = (D + 2)(3D − 1)
AB = 3D2 + 5D − 2
Now,
dy
BAy = (3D − 1) + 2y
dx
d2 y dy
BAy = 3 2 + 5 − 2y
dx dx
ABy = (3D2 + 5D − 2)y
We have,
BA = (3D − 1)(D + 2)
BA = 3D2 + 5D − 2
Dk emk = mk emk
it is easy to find out the effect of the operator on emk . Let f (D) be a polynomial in D,
Then,
f (D)emx = a0 mn emx + a1 mn−1 emx + · · · + an−1 memx + an emx
so that
f (D)emx = emx f (m)
If m is a root of f (m), then it follows
f (D)emx = 0
Now, we consider the effect of the operator (D − a) on the product of eax and the
function y. We have
Using linearity principle we can say that if f (D) is an operator with constant
coefficients, then
eax f (D)y = f (D − a)eax y
Example 2.
Let
f (D) = 2D2 + 5D − 12
. Then the equation f (m) = 0 is
2m2 + 5m − 12 = 0
or
(m + 4)(2m − 3) = 0
of which the roots are m1 = −4 and m2 = 23 .
We can see that
(2D2 + 5D − 12)e−4x = 0
and
3
(2D2 + 5D − 12)e 2 x = 0
3
In other words, y1 = e−4x and y2 = e 2 x are solutions of
(2D2 + 5D − 12)y = 0
Example 3.
(D + 4)4 y = 0
Solution:
Multiply e3x , we have
e3x (D + 4)4 y = 0
By the exponential shift
D4 (e3x y) = 0
Integrating four times,
e3x y = c4 + c3 x + c2 x2 + c1 x3
Finally,
y = (c4 + c3 x + c2 x2 + c1 x3 )e−3x
Bibliography
[Boyce and Diprima, 2001] Boyce, W. and Diprima, R. (2001). Elementary Differential
Equations with Boundary Value Problems. John Wylie & Sons, Inc.
[Bronson and Costa, 2009] Bronson, R. and Costa, G. (2009). Schaum’s Outline of
Differential Equations. McGraw-Hill Education, 3rd edition.
[Lebl, 2019] Lebl, J. (2019). Notes on Diffy Qs: Differential Equations for Engineers.
Independently Published.
[Rainville et al., 2013] Rainville, D., Bedient, P., and Bedient, R. (2013). Elementary
Differential Equations: Pearson New International Edition. Pearson Education, Limited.
[Zill, 2012] Zill, D. (2012). A First Course in Differential Equations with Modeling
Applications. Cengage Learning.
Module 5 Homogeneous Linear
Differential Equations with Constant
Coefficients
Introduction
In this module, we shall learn how to solve higher-order linear differential equation.
This lecture, however, will focus only for those equations with constant coefficients. The
auxiliary equation will be introduced. The roots of the auxiliary equation will determine the
form of solution of the differential equation.
Topic Outcomes
dn y dn−1 y dy
a0 n
+ a 1 n−1
+ · · · + an−1 + an y = 0 (1)
dx dx dx
may be written in the form
f (D)y = 0 (2)
where f(D) is a linear differential operator. As we saw in the previous lecture, if m is any
root of the algebraic equation f (m) = 0, then
f (m) = 0 (3)
is called the auxilliary equation that is associated with equations (1) and (2).
The auxiliary equation for (1) is of degree n. Let its roots be m1 , m2 , · · · , mn . If these
roots are all real and distinct, then the n solutions
are linearly independent, then the general solution of (1) can be written as
Supposed that in (1) the operator f (D) has repeated factors; that is, the auxiliary
equation f (m) = 0 has repeated roots. Then the method discussed before does not yield the
general solution. Let the auxiliary equation have three equal roots m1 = m2 = m3 = b. The
corresponding part of the solution yielded by the method of the previous section is
y = c4 ebx
with c4 = c1 + c2 + c3 . Thus, corresponding to the three roots under consideration, this
method has yielded only the solution (5). The difficulty is present, of course, because the
three solutions corresponding to the roots m1 = m2 = m3 = b are not linearly independent.
The the operator f (D) must have a factor (D − b)n . We wish to find n linearly
independent y’s for which
(D − b)n y = 0 (5)
Turning to the properties of the differential operator and writing m = b, we find that
Furthermore, if f (D) contains the factor (D − b)n , then equation (2) can be written
where g(D) contains all the factors of f (D) except (D − b)n . Then any solution of equation
(5) is also a solution of (8) and hence of (1).
Now we are in a position to write the solution of equation (1) whenever the auxiliary
equation has only real roots. Each root of the auxiliary equation is either distinct from all
the other roots or it is one of a set of equal roots. Corresponding to a root mi distinct from
all others, there is the solution
yi = ci emi x (9)
and corresponding to n equal roots m1 = m2 = · · · = mn = b, there are the solutions
The collection of solutions (10) has the proper number of elements, a number equal to
the order of the differential equation, because there is one solution corresponding to each
root of the auxiliary equation. The solutions thus obtained can be proven to be linearly
independent.
Auxiliary Equations with Complex Roots
Since the auxiliary equation may have imaginary roots, we need now to lay down a
definition of ez for imaginary z. Let z = α + jβ with α and β real. Since it is desirable to
have the ordinary laws of exponents remain valid, it is wise to require that
eα+jβ = eα ejβ (11)
To eα , with α real, we will use the usual meaning. Now, for ejβ , β real. In calculus, we know
that for all real x ∞
X xn
ex = (12)
n=0
n!
If we put tentatively x = jβ in (12) as a definition of ejβ , we get
jβ j 2 β 2 j nβ n
ex = 1 +
+ + ··· + + ··· (13)
1! 2! n!
Separating the even powers of β from the odd powers of β in (13) yields
j 2β 2 j 4β 4 j 2k β 2k jβ j 3 β 3 j 2k+1 β 2k+1
ejβ = 1 + + + ··· + + ··· + + + ··· + + ··· (14)
2! 4! (2k)! 1! 3! (2k + 1)!
or ∞ ∞
jβ
X j 2k β 2k X j 2k+1 β 2k+1
e = + (15)
k=0
(2k)! k=0
(2k + 1)!
But the series on the right in (15) are precisely those for cos β and sin β. Hence we have
ejβ = cos β + j sin β (16)
Combining (16) with (11), we now put forward a reasonable definition of eα+jβ , namely,
eα+jβ = eα (cos β + j sin β) (17)
It is interesting and important that, with the definition (17), the function ez for the
complex z retains many of the properties possessed by the function ex for real x. Here we
need in particular to know that if
y = e(a+jb)x
with a, b, and x real, then it is a solution of
[D − (a + jb)]y = 0
The result desired follows at once by differentiation, with respect to x, of the function
y = eax (cos bx + j sin bx)
Imaginary Roots
Consider a differential equation f (D)y = 0 for which the auxiliary equation f (m) = 0
has real coefficients. From elementary algebra we know that if the auxiliary equation has
any imaginary roots, those roots must occur in conjugate pairs. Thus if,
m1 = a + jb
is a root of the equation f (m) = 0, with a and b real and b 6= 0, then
m2 = a − jb
is also a root of f (m) = 0.
It must be kept in mind that this result is a consequence of the reality of the coefficients
in the equation f (m) = 0. Imaginary roots do not necessarily appear in pairs in an algebraic
equation whose coefficients involve imaginaries.
Finally, we can have c3 + c4 = c1 , and j(c3 − c4 ) = c2 , where c1 and c2 are new arbitrary
constants. Then, the linear equation
f (D)y = 0
with auxiliary equation f (m) = 0 whose roots are m = a ± jb, b 6= 0, have a general solution
y = eax (c1 cos bx + c2 sin bx) .
Case 1. Real and Distinct Roots
Example 4.
Solution:
The auxilliary equation is
m3 + 2m2 − 8m = 0
m(m + 4)(m − 2) = 0
and its roots are m = 0, 2, −4. And the desired solution can be written as
y = c1 + c2 e2x + c3 e−4x
Example 5.
(D2 − D − 6)y = 0
Solution:
The auxilliary equation is
m2 − m − 6 = 0
(m − 3)(m + 2) = 0
and its roots are m = −2, 3. And the desired solution can be written as
y = c1 e−2x + c2 e3x
Example 6.
Solution:
The auxilliary equation is
m3 − 3m2 − m + 3 = 0
(m + 1)(m − 1)(m − 3) = 0
and its roots are m = −1, 1, 3. And the desired solution can be written as
y = c1 e−x + c2 ex + c3 e3x
Example 7.
Solution:
The auxiliary equation is
4m4 − 15m2 + 5m + 6 = 0
(m − 1)(m + 2)(2m + 1)(2m − 3) = 0
1 3
and its roots are m = −2, − , 1, . And the desired solution can be written as
2 2
1 3
y = c1 e−2x + c2 e− 2 x + c3 ex + c4 e 2 x
Example 8.
Solution:
The auxiliary equation is
3 1
and its roots are m = −2, − , − , 4. And the desired solution can be written as
2 2
3 1
y = c1 e−2x + c2 e− 2 x + c3 e− 2 x + C4 e4x
Example 9.
(10D3 + D2 − 7D + 2)y = 0
Solution:
The auxiliary equation is
10m3 + m2 − 7m + 2 = 0
(m + 1)(2m − 1)(5m − 2) = 0
1 2
and its roots are m = −1, , . And the desired solution can be written as
2 5
1 2
y = c1 e−x + c2 e 2 x + c3 e 5 x
Example 10.
D3 − D2 − 4D + 4 y = 0
Solution:
The auxiliary equation is
m3 − m2 − 4m + 4 = 0
(m − 1)(m + 2)(m − 2) = 0
and its roots are m = −2, 1, 2. And the desired solution can be written as
y = c1 e−2x + c2 ex + c3 e2x
Example 11.
Solution:
The auxiliary equation is
4m3 − 13m + 6 = 0
(m + 2)(2m − 1)(2m − 3) = 0
1 3
and its roots are m = −2, , . And the desired solution can be written as
2 2
1 3
y = c1 e−2x + c2 e 2 x + c3 e 2 x
Example 12.
D3 − 5D − 2 y = 0
Solution:
The auxiliary equation is
m3 − 5m − 2 = 0
(m + 2)(m2 − 2m − 1) = 0
√ √
and its roots are m = −2, (1 − 2), (1 + 2). And the desired solution can be written as
√ √
y = c1 e−2x + c2 e(1− 2)x
+ c3 e(1+ 2)x
Example 13.
Solution:
The auxiliary equation is
m3 + 2m2 − 8m = 0
m(m + 4)(m − 2) = 0
and its roots are m = 0, −4, 2, 0. And the desired solution can be written as
y = c1 + c2 e2x + c3 e−4x
Case 2. Real and Repeated Roots
Example 1.
Solve the equation
4D2 − 4D + 1 y = 0
Solution:
The auxiliary equation is
4m2 − 4m + 1 = 0
2
1
m− =0
2
1 1
and its roots are m = , . And the desired solution can be written as
2 2
1
y = e 2 x (c1 + c2 x)
1 1
y = c1 e 2 x + c2 xe 2 x
Example 2.
Solve the equation
D2 + 6D + 9 y = 0
Solution:
The auxiliary equation is
m2 + 6m + 9 = 0
(m + 3)2 = 0
and its roots are m = −3, −3. And the desired solution can be written as
y = e−3x (c1 + c2 x)
y = c1 e−3x + c2 xe−3x
Example 3.
Solve the equation
D3 − 4D2 + 4D y = 0
Solution:
The auxiliary equation is
m3 − 4m2 + 4m = 0
m(m2 − 4m + 4) = 0
m(m − 2)2 = 0
and its roots are m = 0, 2, 2. And the desired solution can be written as
y = c1 + e2x (c2 + c3 x)
y = c1 + c2 e2x + c3 xe2x
Example 4.
Solve the equation
Solution:
The auxiliary equation is
1
and its roots are m = 0, 0, − . And the desired solution can be written as
2
1
y = c1 + c2 x + c3 e− 2 x + c4 e2x
Example 5.
D3 + 3D2 − 4 y = 0
Solution:
The auxiliary equation is
m3 + 3m2 − 4 = 0
(m − 1)(m + 2)2 = 0
and its roots are m = 1, −2, −2. And the desired solution can be written as
y = c1 ex + e−2x (c2 + c3 x)
y = c1 ex + c2 e−2x + c3 xe−2x
Example 6.
D5 − 16D3 y = 0
Solution:
The auxiliary equation is
m5 − 16m3 = 0
m3 (m2 − 16) = 0
and its roots are m = 0, 0, 0, −4, 4. And the desired solution can be written as
y = c1 + c2 x + c3 x2 + c4 e−4x + c5 e4x
Example 7.
Solve the equation
Solution:
The auxiliary equation is
1 1
and its roots are m = −1, −1, , . And the desired solution can be written as
2 2
1
y = e−x (c1 + c2 x) + e 2 x (c3 + c4 x)
1 1
y = c1 e−x + c2 xe−x + c3 e 2 x + c4 xe 2 x
Example 8.
Solve the equation
Solution:
The auxiliary equation is
and its roots are m = −2, −2, −2, 3. And the desired solution can be written as
Solution:
The auxiliary equation is
1 1
and its roots are m = 3, −1, −1, − , − . And the desired solution can be written as
2 2
1
y = c1 e3x + e−x (c2 + c3 x) + e− 2 x (c4 + c5 x)
1 1
y = c1 e3x + c2 e−x + c3 xe−x + c4 e− 2 x + c5 xe− 2 x
Example 10.
Solve the equation
D5 − 5D4 + 7D3 + D2 − 8D + 4 y = 0
Solution:
The auxiliary equation is
m5 − 5m4 + 7m3 + m2 − 8m + 4 = 0
(m + 1)(m − 1)2 (m − 2)2 = 0
and its roots are m = −1, 1, 1, 2, 2. And the desired solution can be written as
D2 − 2D + 5 y = 0
Solution:
The auxiliary equation is
m2 − 2m + 5 = 0
and its roots are m = 1 ± j2. And the desired solution can be written as
Example 2.
D2 − 2D + 2 y = 0
Solution:
The auxiliary equation is
m2 − 2m + 2 = 0
and its roots are m = 1 ± j. And the desired solution can be written as
D2 + 9 y = 0
Solution:
The auxiliary equation is
m2 + 9 = 0
and its roots are m = ±j3. And the desired solution can be written as
Example 4.
D2 + 6D + 13 y = 0
Solution:
The auxiliary equation is
m2 + 6m + 13 = 0
and its roots are m = −3 ± j2. And the desired solution can be written as
D2 − 4D + 7 y = 0
Solution:
The auxiliary equation is
m2 − 4m + 7 = 0
√
and its roots are m = 2 ± j 3. And the desired solution can be written as
√ √
y = e2x (c1 cos 3x + c2 sin 3x)
Example 6.
D3 + 2D2 + D + 2 y = 0
Solution:
The auxiliary equation is
m3 + 2m2 + m + 2 = 0
(m + 2)(m2 + 1) = 0
and its roots are m = −2, ±j. And the desired solution can be written as
D4 + 2D3 + 10D2 y = 0
Solution:
The auxiliary equation is
m4 + 2m3 + 10m2 = 0
m2 (m2 + 2m + 10) = 0
√
and its roots are m = 0, 0, −1 ± j 3. And the desired solution can be written as
√ √
y = c1 + c2 x + e−x (c3 cos 3x + c4 sin 3x)
Example 8.
D3 + 8 y = 0
Solution:
The auxiliary equation is
m3 + 8 = 0
(m + 2)(m2 − 2m + 4) = 0
√
and its roots are m = −2, 1 ± j 3. And the desired solution can be written as
2D3 − D2 + 36D − 18 y = 0
Solution:
The auxiliary equation is
2m3 − m2 + 36m − 18 = 0
(2m − 1)(m2 + 18) = 0
1 √
and its roots are m = , ±j3 2. And the desired solution can be written as
2
1 √ √
y = c1 e 2 x + c2 cos 3 2x + c3 sin 3 2x
Example 10.
D2 D2 + 4 D3 − 27 y = 0
Solution:
The auxilliary equation is
m2 m2 + 4 m3 − 27 = 0
m2 m2 + 4 m − 3 m2 + 3m + 9 = 0
√
−3 ± 3i 3
and its roots are m = 0, 0, 3, ±2i, . And the desired solution can be written as
2
√ √
3 3 3 3 3
y = c1 + c2 x + c3 e3x + c4 cos 2x + c5 sin 2x + e− 2 x c6 cos x + c7 sin x
3 3
Case 4. Repeated Imaginary Roots
Example 1.
Solve the equation
2
D2 + 5 y = 0
Solution:
The auxiliary equation is
(m2 + 5)2 = 0
(m2 + 5)(m2 + 5) = 0
√ √
and its roots are m = ±j 5, ±j 5. And the desired solution can be written as
√ √ √ √
y = c1 cos 5x + c2 sin 5x + c3 x cos 5x + c4 x sin 5x
√ √
y = (c1 + c3 x) cos 5x + (c2 + c4 x) sin 5x
Example 2.
Solve the equation
D4 + 18D2 + 81 y = 0
Solution:
The auxiliary equation is
m4 + 18m2 + 81 = 0
(m2 + 9)2 = 0
and its roots are m = ±j3, ±j3. And the desired solution can be written as
y = c1 cos 3x + c2 sin 3x + c3 x cos 3x + c4 x sin 3x
y = (c1 + c3 x) cos 3x + (c2 + c4 x) sin 3x
Example 3.
Solve the equation
D4 + 2D2 + 1 y = 0
Solution:
The auxiliary equation is
m4 + 2m2 + 1 = 0
(m2 + 1)2 = 0
and its roots are m = ±j, ±j. And the desired solution can be written as
Example 4.
Solve the equation
D4 + 13D2 + 36 y = 0
Solution:
The auxiliary equation is
m4 + 13m2 + 36 = 0
(m2 + 6)2 = 0
√ √
and its roots are m = ±j 6, ±j 6. And the desired solution can be written as
√ √ √ √
y = c1 cos ( 6x) + c2 sin ( 6x) + c3 x cos ( 6x) + c4 x sin ( 6x)
√ √
y = (c1 + c3 x) cos ( 6x) + (c2 + c4 x) sin ( 6x)
Example 5.
Solve the equation
D6 + 9D4 + 24D2 + 16 y = 0
Solution:
The auxiliary equation is
m6 + 9m4 + 24m2 + 16 = 0
(m2 + 1)(m2 + 4)2 = 0
and its roots are m = ±j, ±j2, ±j2. And the desired solution can be written as
y = c1 cos x + c2 sin x + c3 cos 2x + c4 sin 2x + c5 x cos 2x + c6 x sin 2x
y = c1 cos x + c2 sin x + (c3 + c5 x) cos 2x + (c4 + c6 x) sin 2x
Example 6.
Solve the equation
D7 + 9D5 + 24D3 + 16D y = 0
Solution:
The auxiliary equation is
m7 + 9m5 + 24m3 + 16m = 0
m(m2 + 1)(m2 + 4)2 = 0
and its roots are m = 0, ±j, ±j2, ±j2. And the desired solution can be written as
y = c0 + c1 cos x + c2 sin x + c3 cos 2x + c4 sin 2x + c5 x cos 2x + c6 x sin 2x
y = c0 + c1 cos x + c2 sin x + (c3 + c5 x) cos 2x + (c4 + c6 x) sin 2x
Example 7.
D6 + 3D4 + 3D2 + 1 y = 0
Solution:
The auxiliary equation is
m6 + m4 + 3m2 + 1 = 0
(m2 + 1)3 = 0
and its roots are m = ±j, ±j, ±j. And the desired solution can be written as
Example 8.
D − 1 D4 − 9 y = 0
Solution:
The auxiliary equation is
(m − 1)(m4 − 9) = 0
(m − 1)(m2 + 3)(m2 − 3) = 0
√ √
and its roots are m = 1, ± 3, ±j 3. And the desired solution can be written as
√
y = c1 ex + e 3x
(c2 + c3 x) + (c4 cos 3x + c5 sin 3x)
Example 9.
D3 − 8 D4 − 4D2 y = 0
Solution:
The auxiliary equation is
√
and its roots are m = 0, 0, 2, 2, −2, −1 ± j 3. And the desired solution can be written as
√ √
y = c1 + c2 x + e2x (c3 + c4 x) + c5 e−2x + e−x c6 cos ( 3x) + c7 sin ( 3x)
√ √
y = c1 + c2 x + c3 e2x + c4 xe2x + c5 e−2x + c6 e−x cos ( 3x) + c7 e−x sin ( 3x)
Example 10.
D5 − D2 D2 + 10D − 40 y = 0
Solution:
The auxiliary equation is
√
1±j 3 √
and its roots are m = 0, 0, 1, − , −5 ± 65. And the desired solution can be written
2
as
√ √
√ √
x (−5+ 65)x (−5− 65)x − 12 x 3 3
y = c1 + c2 x + c3 e + c4 e + c5 e +e c6 cos x + c7 sin x
2 2
Bibliography
[Boyce and Diprima, 2001] Boyce, W. and Diprima, R. (2001). Elementary Differential
Equations with Boundary Value Problems. John Wylie & Sons, Inc.
[Bronson and Costa, 2009] Bronson, R. and Costa, G. (2009). Schaum’s Outline of
Differential Equations. McGraw-Hill Education, 3rd edition.
[Lebl, 2019] Lebl, J. (2019). Notes on Diffy Qs: Differential Equations for Engineers.
Independently Published.
[Rainville et al., 2013] Rainville, D., Bedient, P., and Bedient, R. (2013). Elementary
Differential Equations: Pearson New International Edition. Pearson Education, Limited.
[Zill, 2012] Zill, D. (2012). A First Course in Differential Equations with Modeling
Applications. Cengage Learning.
Module 6 Non-Homogeneous Linear
Differential Equations
Introduction
Topic Outcomes
dn y dn−1 y dy
b0 (x) n
+ b 1 (x) n−1
+ · · · + bn−1 (x) + bn (x)y = R(x)
dx dx dx
From the previous lecture it is found out that the general solution of a non-homogeneous
equation is
y = yc + yp
where yc is the complementary function corresponding to the solution of the homogeneous
equation
dn y dn−1 y dy
b0 (x) n + b1 (x) n−1 + · · · + bn−1 (x) + bn (x)y = 0
dx dx dx
and yp is the particular solution that takes on the form of R(x).
The table shown, shows the forms of the particular solution for the different forms of
R(x).
where: s is the smallest non-negative integer that will make yp (x) not of the same form as
the corresponding homogeneous equation.
NOTE : This method is applicable only if R(x) is of the form similar to the solutions
of a constant coefficient homogeneous linear differential equation.
Example 11.
Solution:
The auxiliary equation is
m−4=0
yc = c1 e4x
yp = A
(D − 4)yp = 5
Dyp − 4yp = 5
Since yp = A, then
Dyp − 4yp = 5
DA − 4A = 5
5
Solving the equation we have A = − Therefore
4
5
yp = −
4
Hence the general solution of (1) is given by,
5
y = c1 e4x −
4
Example 12.
Solution:
The auxiliary equation is
m2 + m = 0
m(m + 1) = 0
yc = c1 + c2 e−x
yp = A sin x + B cos x
yp = A sin x + B cos x
Dyp = A cos x − B sin x
D2 yp = −A sin x − B cos x
1 1
Using equating coefficients we have A = − and B = − .Therefore
2 2
1 1
yp = sin x + cos x
2 2
Hence the general solution of (1) is given by,
1 1
y = c1 + c2 e−x + sin x + cos x
2 2
Example 13.
Solution:
The auxiliary equation is
m2 − 4m + 4 = 0
(m − 2)2 = 0
yc = c1 e2x + c2 xe2x
yp = Aex
yp = Aex
Dyp = Aex
D2 yp = Aex
Aex − 4Aex + 4 = ex
yp = ex
y = c1 e2x + c2 xe2x + ex
Example 14.
Solution:
The auxiliary equation is
m2 − 3m + 2 = 0
(m − 2)(m − 1) = 0
yc = c1 ex + c2 e2x
yp = A + Bx + Cx2
yp = A + Bx + Cx2
Dyp = B + 2Cx
D2 yp = 2C
yp = 4 + 3x + x2
y = c1 ex + c2 e2x + x2 + 3x + 4
Example 15.
Solution:
The auxiliary equation is
m2 + 3m + 2 = 0
(m + 2)(m + 1) = 0
yc = c1 e−2x + c2 e−x
yp = A sin 3x + B cos 3x
yp = A sin 3x + B cos 3x
Dyp = 3A cos 3x − 3B sin 3x
D2 yp = −9A sin 3x − 9B cos 3x
−9A sin 3x − 9B cos 3x + 3(3A cos 3x − 3B sin 3x) + 2(A sin 3x + B cos 3x) = 2 sin 3x
(−7A − 9B) sin 3x + (9A − 7B) cos 3x = 2 sin 3x
7 9
Using equating coefficients we have A = − and B = − . Therefore
65 65
7 9
yp = − sin 3x − cos 3x
65 65
Hence the general solution of (1) is given by,
7 9
y = c1 e−2x + c2 e−x − sin 3x − cos 3x
65 65
Example 16.
Solution:
The auxiliary equation is
m2 − 3m + 2 = 0
(m − 2)(m − 1) = 0
yc = c1 ex + c2 e2x
yp = A + Bx + Cx2 + Ex3
Since
yp = A + Bx + Cx2 + Ex3
Dyp = B + 2Cx + 3Ex2
D2 yp = 2C + 6Ex
y p = x3
y = c1 ex + c2 e2x + x3
Example 17.
Solution:
The auxiliary equation is
m2 + 4 = 0
yc = c1 cos 2x + c2 sin 2x
yp = A + Bx + Cex
yp = A + Bx + Cex
Dyp = B + Cex
D2 yp = Cex
yp = −x + ex
y = c1 cos 2x + c2 sin 2x − x + ex
Example 18.
Solution:
The auxiliary equation is
m2 + 4 = 0
yc = c1 cos 2x + c2 sin 2x
yp = A + Bx + Cx2 + Eex
yp = A + Bx + Cx2 + Eex
Dyp = B + 2Cx + Eex
D2 yp = 2C + Eex
1
Solving the equation we have A = , B = 0, C = −1, E = 1 Therefore
2
1
yp = − x2 + e x
2
Hence the general solution of (1) is given by,
1
y = c1 cos 2x + c2 sin 2x + − x2 + e x
2
Example 19.
Solution:
The auxiliary equation is
m2 − 9 = 0
(m − 3)(m + 3) = 0
yc = c1 e−3x + c2 e3x
yp = Ae2x
yp = Ae3x
Dyp = 2Ae2x
D2 yp = 4Ae2x
4
Solving the equation we have A = − Therefore
5
4
yp = − e2x
5
Hence the general solution of (1) is given by,
4
y = c1 e−3x + c2 e3x − e2x
5
Example 20.
Solution:
Rewrite (1) as
(D2 − 3D − 4)y = 6ex (2)
The auxiliary equation is
m2 − 3m − 4 = 0
(m − 4)(m + 1) = 0
yc = c1 e−x + c2 xe4x
yp = Aex
yp = Aex
Dyp = Aex
D2 yp = Aex
yp = −ex
y = c1 e−x + c2 xe4x − ex
Particular Solution by Inspection
Example 1.
Solve the equation
(D2 + 4)y = 12 (1)
Solution:
The auxiliary equation is
m2 + 4 = 0
and its roots are m = ±j2. Hence the complimentary function is given by
yc = c1 cos 2x + c2 sin 2x
Solution:
The auxiliary equation is
m2 + 4m + 4 = 0
(m + 2)2 = 0
and its roots are m = −2, −2. Hence the complimentary function is given by
yc = c1 e−2x + c2 e−2x
Example 3.
Solve the equation
(D2 + 4D)y = 12 (1)
Solution:
The auxiliary equation is
m2 + 4m = 0
m(m + 4) = 0
and its roots are m = 0, −4. Hence the complimentary function is given by
yc = c1 + c2 e−4x
Solution:
The auxiliary equation is
m3 − 3m + 2 = 0
(m − 1)2 (m + 2) = 0
and its roots are m = 1, 1, −2. Hence the complimentary function is given by
yc = c1 ex + c2 xex + c3 e−2x
Example 5.
Solve the equation
(D4 − 4D2 )y = 24 (1)
Solution:
The auxiliary equation is
m4 − 4m2 = 0
(m)2 (m2 − 4) = 0
and its roots are m = 0, 0, −2, 2. Hence the complimentary function is given by
yc = c1 + c2 x + c3 e−2x + c4 e2x
Solution:
The auxiliary equation is
m5 − m3 = 0
m3 (m2 − 1) = 0
and its roots are m = 0, 0, 0, −1, 1. Hence the complimentary function is given by
yc = c1 + c2 x + c3 x2 + c4 e−x + c5 ex
24x3
yp = = −4x3
6(−1)
y = c1 + c2 x + c3 x2 + c4 e−x + c5 ex − 4x3
Example 7.
Solve the equation
(D2 + 9)y = 18 (1)
Solution:
The auxiliary equation is
m2 + 9 = 0
and its roots are m = ±j3. Hence the complimentary function is given by
yc = c1 cos 3x + c2 sin 3x
Example 8.
Solve the equation
(D4 + 4D2 + 4)y = −20 (1)
Solution:
The auxiliary equation is
m4 + 4m2 + 4 = 0
(m2 + 2)2 = 0
√
and its roots are m = ±j 2. Hence the complimentary function is given by
√ √
yc = c1 cos 2x + c2 sin 2x
By inspection a particular solution of (1) is
−20
yp = = −5
4
Therefore the general solution of (1) is
√ √
y = c1 cos 2x + c2 sin 2x − 5
Example 9.
Solve the equation
(D5 − 9D3 )y = 27 (1)
Solution:
The auxiliary equation is
m5 − 9m3 = 0
m3 (m2 − 9) = 0
and its roots are m = 0, 0, 0, −3, 3. Hence the complimentary function is given by
yc = c1 + c2 x + c3 x3 + c4 e−3x + c5 e3x
By inspection a particular solution of (1) is
27x3 1
yp = = − x3
6(−9) 2
Therefore the general solution of (1) is
1
y = c1 + c2 x + c3 x3 + c4 e−3x + c5 e3x − x3
2
Example 10.
Solve the equation
(D4 + D2 )y = −12 (1)
Solution:
The auxiliary equation is
m4 + m2 = 0
m2 (m2 + 1) = 0
and its roots are m = 0, 0, ±j. Hence the complimentary function is given by
yc = c1 + c2 x + c3 cos x + c4 sin x
By inspection a particular solution of (1) is
−12x2
yp = = −6x2
2(1)
Therefore the general solution of (1) is
y = c1 + c2 x + c3 cos x + c4 sin x − 6x2
Reduction Order
This method proposed by D’Alembert is not only applicable to equations with constant
coefficient but also to equations with variable coefficients. However, we did not deal with
such equations for the method of solving the complementary functions of such equations
involves a more complicated approach.
y 00 + P (x)y + Q(x)y = 0
We want another solution that is linearly independent with the other solution. Let y2
be the solution corresponding to R(x) and y1 and y2 are linearly independent, then their
y2 y2 (x)
quotient is nont a constant on the interval, i.e., = u(x) or y2 (x) = u(x)y1 (x).
y1 y1 (x)
The function u(x) can be found by substituting y2 (x) = u(x)y1 (x) into the given differential
equation. This method is called reduction of order because we must solve a linear first-order
differential equation to find u.
y20 = y1 u0 + uy10
y200 = y1 u00 + 2y10 u0 + uy100
y 00 + P y 0 + Qy = R
We have
y1 u00 + 2y10 u0 + uy100 + (y1 u0 + uy10 ) · P + y1 u · Q = R.
Combining all u’s
y1 u00 + (2y10 + P y1 )u0 + (y100 + P y10 + Qy1 )u = R
But y1 is a solution of y 00 + P y + Qy = 0, hence we will have
Using the methods discussed before, we can solve for w. Then form u0 = w we can get
u and finally we can have the other solution y2 = uy1
Note that the method is not restricted to equations constant coefficients. It depends
only upon our knowing a single nonzero solution of equation. For practical purposes, the
method depends also upon our being able to effect the integration.
Example 1.
(D2 − 1)y = x − 1 (1)
Solution:
yc = c1 ex + c2 e−x
Let
yp = vex
Dyp = vex + v 0 ex
D00 yp = vex + v 0 ex + v 0 ex + v 00 ex
D00 yp = v 00 ex + 2vex + vex
Substitute directly to (1) when y = yp
v 00 ex + 2vex + vex − vex = x − 1
v 00 ex + 2v 0 ex = x − 1
Let v 0 = ω, then
v 00 ex + 2v 0 ex = x − 1
ω 0 ex + 2ωex = x − 1
1 0 x x 1
x
ω e + 2ωe = x − 1 x
e e
x−1
ω 0 + 2ω =
ex
dω
+ 2ω = e−x (x − 1)
dx
We now have,
dω + 2ωdx = e−x (x − 1)dx (2)
Solving for the integrating factor we have
R
2dx
e = e2x
And therefore
y = c1 ex + c2 e−x − x + 1
Example 2.
(D2 − 5D + 6)y = 2ex (1)
Solution:
yc = c1 ex + c2 e−x
Let
yp = ve2x
Dyp = 2ve2x + v 0 e2x
D00 yp = v 00 e2x + 4v 0 e2x + 4ve2x
Let v 0 = ω, then
ω
= −4e−2x
ex
Solving for ω we have
ω = −4e−x
Since v 0 = ω, then
dv
=ω
dx
dv = −4e−x dx
Z Z
dv = −4 e−x dx
v = 4e−x
yp = 4e−x e2x
yp = 4ex
And therefore
y = c1 ex + c2 e−x + 4ex
Example 3.
(D2 + 1)y = sec x (1)
Solution:
yc = c1 cos x + c2 sin x
Let
yp = v sin x
Dyp = v cos x + v 0 sin x
D00 yp = v 00 sin x + 2v 0 cos x − v sin x
Let v 0 = ω, then
ω = − csc2 x ln | cos x|
Since v 0 = ω, then
dv
=ω
dx
dv = − csc2 x ln | cos x|dx
Z Z
dv = − csc2 x ln | cos x|dx
v = cot x ln | cos x| − x
Now since yp = v sin x, and v = cot x ln | cos x| − x, then
yp = (cot x ln | cos x| − x) sin x
yp = cos x ln | cos x| − x sin x
And therefore
y = c1 cos x + c2 sin x + cos x ln | cos x| − x sin x
Example 4.
(D2 + 1)y = sec3 x (1)
Solution:
yc = c1 cos x + c2 sin x
Let
yp = v cos x
Dyp = −v sin x + v 0 cos x
D00 yp = v 00 cos x − 2v 0 sin x − v cos x
Substitute directly to (1) when y = yp
v 00 cos x − 2v 0 sin x − v cos x + v cos x = sec3 x
v 00 cos x − 2v 0 sin x = sec3 x
Let v 0 = ω, then
v 00 cos x − 2v 0 sin x = sec3 x
ω 0 cos x − 2ω sin x = sec3 x
ω 0 − 2ω tan x = sec4 x
dω
− 2ω tan x = sec4 x
dx
We have
dω − 2ω tan xdx = sec4 xdx (2)
Solving for the integrating factor we have
R
e− 2 tan xdx
= e−2 ln | cos x| = cos2 x
ω = tan x sec2 x
Since v 0 = ω, then
dv
=ω
dx
dv = tan x sec2 xdx
Z Z
dv = tan x sec2 xdx
1
v= tan2 x
2
Now since yp = v cos x, and v = 12 tan2 x, then
1 1
yp = tan2 x cos xyp = sec x sin2 x
2 2
And therefore
1
y = c1 cos x + c2 sin x + sec x sin2 x
2
Example 5.
Solution:
yc = c1 cos x + c2 sin x
Let
yp = v sin x
Dyp = v cos x + v 0 sin x
D00 yp = v 00 sin x + 2v 0 cos x − v sin x
Let v 0 = ω, then
ω = csc2 x ln | sin x|
Since v 0 = ω, then
dv
=ω
dx
dv = csc2 x ln | sin x|dx
Z Z
dv = csc2 x ln | sin x|dx
And therefore
yc = c1 y1 + c2 y2 (3)
u1 (y100 + P y10 + Qy)+u2 (y200 + P y20 + Qy)+y1 u001 +y10 u01 +y2 u002 +y20 u02 +P (u01 y1 + u02 y2 )+y10 u01 +y20 u02 = R
d
From here (y100 + P y10 + Qy) = 0 and (y200 + P y20 + Qy), while y1 u001 + y10 u01 = (y1 u01 ) and
dx
d
y2 u002 + y20 u02 = (y2 u02 )
dx
Now we have,
d
y1 u01 + y2 u02 + y1 u01 + y2 u02 P + y10 u01 + y20 u02 = R
(5)
dx
comparing the left and the right side og the equation, we will have a system of
y1 u01 + y2 u02 = 0
y10 u01 + y20 u02 = R
By Cramer’s Rule
y 1 y 2 0 y2 y 1 0
W = 0
W1 =
W2 = 0
y1 y20 R y20 y1 R
We can solve for u01 and u02
W1 y2 R
u01 ==−
W W
0 W2 y1 R
u2 = =
W W
The functions u1 and u2 can be found by integration. The determinant W is recognized as
the Wronskian of y1 and y2 . By linear independence on I, we know that W (y1 , y2 ) 6= 0 for
every real x.
Example 1.
(D2 − 1)y = ex + 1 (1)
Solution:
yc = c1 ex + c2 e−x
yp = Aex + Be−x
y 0 = Aex + A0 ex − Be−x + B 0 e−x
Let
A0 ex + B 0 e−x = 0 (2)
yc = c1 cos x + c2 sin x
yp = A cos x + B sin x
y 0 = −A sin x + A0 cos x − B cos x + B 0 sin x
Let
A0 cos x + B 0 sin x = 0 (2)
B 0 = cot x
B 0 = cot x
B = ln | sin x|
Using (2) substitute B 0 to solve A0 , yields
Example 3.
yc = c1 cos x + c2 sin x
yp = A cos x + B sin x
yp0 = −A sin x + A0 cos x − B cos x + B 0 sin x
Let
A0 cos x + B 0 sin x = 0 (2)
B 0 = cot2 x
B 0 = cot2 x
B = − cot x − x
A0 cos x + B 0 sin x = 0
A0 cos x + cot2 x sin x = 0
A0 cos x = cot2 x sin x
A0 = − cot x
A = − ln | sin x|
Solution:
yc = c1 ex + c2 e−x
yp = Aex + Be−x
y 0 = Aex + A0 ex − Be−x + B 0 e−x
Let
A0 ex + B 0 e−x = 0 (2)
A0 ex − B 0 e−x = x − 1 (3)
Solving (2) and (3) simultaneously to get
x−1
A0 =
2ex
Solving for A we have
x−1
A0 = x
2e
Z
1
A0 = e−x (x − 1)dx
2
Z
0 1
A = (xe−x − e−x )dx
2
1
A = − (xe−x )
2
Using (2) substitute A0 to solve B 0 , yields
x−1 x
x
e + B 0 e−x = 0
2e
x−1
B 0 e−x = −
2
0 x−1
B = − −x
Z2e
x−1
B0 = −
2e−x
1
B = − xex + ex
2
1 1
Since A = − (xe−x ) and B = − xex + ex , then
2 2
1 −x x 1 x
yp = (xe )e + − xe + e e−x
x
2 2
yp = −x + 1
y =1 ex + c2 e−x − x + 1
Example 5.
Solution:
yc = c1 cos x + c2 sin x
yp = A cos x + B sin x
y 0 = −A sin x + A0 cos x + B cos x + B 0 sin x
Let
A0 cos x + B 0 sin x = 0 (2)
B 0 = sec2 x
B = tan x
[Boyce and Diprima, 2001] Boyce, W. and Diprima, R. (2001). Elementary Differential
Equations with Boundary Value Problems. John Wylie & Sons, Inc.
[Bronson and Costa, 2009] Bronson, R. and Costa, G. (2009). Schaum’s Outline of
Differential Equations. McGraw-Hill Education, 3rd edition.
[Lebl, 2019] Lebl, J. (2019). Notes on Diffy Qs: Differential Equations for Engineers.
Independently Published.
[Rainville et al., 2013] Rainville, D., Bedient, P., and Bedient, R. (2013). Elementary
Differential Equations: Pearson New International Edition. Pearson Education, Limited.
[Zill, 2012] Zill, D. (2012). A First Course in Differential Equations with Modeling
Applications. Cengage Learning.
Module 7 The Laplace Transform
Introduction
In the introductory calculus, Differential and Integral Calculus, you learned that
differentiation and integration are transforms; this means, roughly speaking, that these
operations transform a function into another function. In this module, we will discuss the
Laplace transform its properties as well as how it can be used to solve initial value
problems of ordinary differential equation.
Topic Outcomes
1. Use the definition of the Laplace transform to derive the transforms of several
elementary functions.
2. Solve for the Laplace transforms of functions using some derived equations.
3. Use the operational properties of the Laplace transform in finding transforms of several
combined functions.
4. Solve initial value problems by applying the Laplace and the inverse Laplace transform.
Definition of the Laplace Transform
is called the Laplace Transform of f , provided that the integral converges. This was named
in honor of the French mathematician Pierre-Simon Marquis de Laplace (1749–1827).
When the defining integral (1) converges, the result is a function of s. In this lecture we
shall use a lowercase letter to denote the function being transformed and the corresponding
capital letter to denote its Laplace transform, e.g.
Furthermore, the given function f (t) in equation (1) is called the inverse transform of
F (s) and is denoted by L −1 {F (s)}.
Solution:
F (s) = L {f (t)}
Z ∞
F (s) = e−st dt
0
∞
1 −st
F (s) = − e
s 0
1
F (s) =
s
Example 2. Transform of t
Evaluate L {t}
Solution:
Z ∞
L {f (t)} = e−st t dt
0
∞
−te−st 1 ∞ −st
Z
L {f (t)} = + e dt
s 0 s 0
1
L {f (t)} = +
s
1 1
L {f (t)} =
s s
1
L {f (t)} =
s2
Solution:
F (s) = L {f (t)}
Z ∞
F (s) = eat e−st dt
Z0 ∞
F (s) = e−(s−a)t dt
0
∞
1 −(s−a)t
F (s) = e when, s − a > 0
s−a
0
1
F (s) =
s−a
The Laplace Transform is a linear operator, i.e., for any function f (t) and g(t) whose
transform exists as F (s) and G(s), respectively. Say the that a and b are constants, then
L {a f (t) + b g(t)} = a L {f (t)} + b L {g(t)}
L {a f (t) + b g(t)} = a F (s) + b G(s)
Example 4. Transform of a Hyperbolic Function
Solve for F (s) and G(s) if f (t) = coshat and g(t) = sinh at when t ≥ 0 and a is a constant.
Solution:
1 1
Since cosh at = eat + e−at and sinh at = eat − e−at ,
2 2
F (s) = L {cosh at}
1 at
F (s) = L −at
e +e
2
1 1
F (s) = L {eat } + L {e−at }
2Z 2
1 ∞ at −st 1 ∞ −at −st
Z
F (s) = e e dt + e e dt
2 0 2 0
1 1 1
F (s) = +
2 s−a s+a
s
F (s) = 2
s − a2
For G(s)
Solution by Calculus:
Let f (t) = cos ωt
Now, Z ∞
1 ω
F (s) = − e−st sin ωt dt (3)
s s 0
Now, Z ∞
ω
G(s) = e−st cos ωt dt (4)
s 0
1 ω ∞ −st
Z
F (s) = − e sin ωt dt
s s 0
1 ω
F (s) = − G(s)
s s
1 ω hω i
F (s) = − F (s)
s s s
s
F (s) = 2
s + ω2
Now for G(s)
ω
G(s) = F (s)
s
ω s
G(s) =
s s2 + ω 2
ω
G(s) = 2
s + ω2
F (s) = L {ejωt }
1
F (s) =
s − jω
s ω
F (s) = 2 2
+j 2
s +ω s + ω2
Now, since f (t) = ejωt cos ωt + j sin ωt=let
As derived before,
s ω
F (s) = +j 2
s2 +ω 2 s + ω2
We can conclude that
s ω
F1 (s) = F2 (s) =
s2 + ω2 s2 + ω2
thus,
F1 (s) = L {f1 (t)} = L {cos ωt} F2 (s) = L {f2 (t)} = L {sin ωt}
Example 6. Transform of a Piece-wise Continuous Function
1
1
s
1
t
s2
2!
t2
s3
n!
tn (n = 0, 1, 2, ...)
sn+1
Γ(a + 1)
ta (a > −1)
sa+1
1
eat
s−a
s
cos ωt
s2 + ω2
ω
sin ωt
s2 + ω2
s
cosh at
s 2 − a2
a
sinh at
s2 − a2
The function Γ(a + 1) is called the gamma function defined by the integral
Z ∞
Γ(a + 1) = e−x xa dx.
0
Also,
Γ(a + 1) = a Γ(a)
For integer values of n ≥ 0
Γ(n + 1) = n!
Example 1.
Obtain
L {t2 + 4t − 5}
Solution:
Example 2.
Obtain
L {t3 − t2 + 4t}
Solution:
Example 3.
Obtain
L {e−2t + 4e−3t }
Solution:
Obtain
L {et+1 }
Solution:
L {et+1 } = L {et · e}
L {et+1 } = e · L {et }
1
L {et+1 } = e ·
s−1
e
L {e } =
t+1
s−1
Example 5.
Obtain
L {5 sin 2t}
Solution:
Obtain
L {5 sin 2t + cos 2t}
Solution:
Solution:
F (s) = L {sin 2t cos 2t}
1
F (s) = L sin 4t
2
1
F (s) = L {sin 4t}
2
1 4
F (s) = · 2
2 s + 42
2
F (s) = 2
s + 16
Example 9.
√
3 1
Solve for F (s) for f (t) = t , use the fact that Γ
2 = π.
2
Solution:
1
F (s) = L {t− 2 }
1
Γ − +1
2
F (s) = − 12 +1
s
1
Γ
2
F (s) = 1
√s
2
πs
F (s) =
s
Inverse Laplace Transform
If F (s) represents the Laplace Transform of a function f (t), i.e., L −1 {f (t)} = F (s),
we say that f (t) is the inverse Laplace transform of F (s). It is defined by the integral
Z σ+j∞
1
L {F (s)} =
−1
F (s)est ds = f (t)u(t) (1)
j2π σ−j∞
where, (
1 t≥0
u(t) =
0 t<0
is the unit step function. Multiplying f (t) to u(t) yields a time function that is zero for t > 0.
Using equation (1) it is possible to derived some specific cases of related f (t) and F (s) and
create a table out of it. If we use this table, we do not have to resort using equation (1) that
requires complex integration to find f (t) given any F (s).
F (s) f (t)
1 δ(t)
1
u(t)
s
1
tu(t)
s2
n!
tn u(t)
sn+1
1
eat u(t)
s−a
s
cos ωt u(t)
s2 + ω2
ω
sin ωt u(t)
s2 + ω2
where, (
∞ t=0
δ(t) =
0 t 6= 0
In evaluating inverse transforms, it often happens that a function of s under
consideration does not match exactly the form of a Laplace transform F (s) given in the
table. It may be necessary to “fix up” the function of s by multiplying and dividing by an
appropriate constant.
Example 1.
Evaluate
4
L −1
s−5
Solution:
4 1 −1 1
L −1
= L
s−5 4 s−5
4 1 5t
L −1
= e u(t)
s−5 4
Example 2.
Evaluate
1
L −1
s5
Solution:
1 1 −1 4!
L −1
= L
s5 4! s4+1
1 1 4
L −1
= t u(t)
s5 24
Example 3.
Evaluate
1
L −1
2
s +9
Solution:
1 1 −1 3
L −1
= L
s2 + 9 3 s2 + 3 2
1 1
L −1
= sin 3t u(t)
s2 + 9 3
Linearity of the Inverse Laplace Transform
The inverse Laplace transform is a linear operator, i.e., for any function F (s) and
G(s) whose inverse transform exists as f (t) and g(t), respectively. Say the that a and b are
constants, then
Example 4.
Evaluate
6 − 2s
L −1
s2 + 4
Solution:
6 − 2s 6 2s
L −1
=L −1
−
s2 + 4 s2 + 4 s2 + 4
−1 6 − 2s 2 s
L = 3L −1
−2 2
s2 + 4 s2 + 2 2 s + 22
6 − 2s
L −1 2 = 3 sin 2t u(t) − 2 cos 2t u(t)
s +4
Example 5.
Evaluate
s2 + 6s + 9
L −1
(s − 1)(s − 2)(s + 4)
Solution: We can use partial fractions expansion.
s2 + 6s + 9
−1 −16/5 25/6 1/30
L −1
=L + +
(s − 1)(s − 2)(s + 4) s−1 s−2 s+4
s2 + 6s + 9
16 −1 1 25 −1 1 1 −1 1
L −1
=− L + L + L
(s − 1)(s − 2)(s + 4) 5 s−1 6 s−2 30 s+4
2
s + 6s + 9 16 25 1
L −1 = − et u(t) + e2t u(t) + e−t u(t)
(s − 1)(s − 2)(s + 4) 5 6 30
Transforms of Derivatives
The goal of this particular topic is for us to be able to use Laplace transform in solving
differential equation. To that purpose we nee to be able to evaluate quantities such as
2
dy dy
L and L .
dt dt2
For example, say f 0 (t) is continuous for the interval t ≥ 0, using the definition of the Laplace
transform we have,
Z ∞
L {f (t)} =
0
e−st f 0 (t)dt
0
∞ Z ∞
L {f (t)} = e f (t) + s
0 −st
e−st f (t)dt
0 0
L {f (t)} = −f (0) + s L {f (t)}
0
or
L {f 0 (t)} = sF (s) − f (0) (1)
The assumption here is that e−st f (t) → 0 as t → ∞. Similarly, we have
Z ∞
L {f (t)} =
00
e−st f 00 (t)dt
0
∞ Z ∞
L {f (t)} = e f (t) + s
00 −st 0
e−st f 0 (t)dt
0 0
L {f (t)} = −f (0) + s L {f (t)}
00 0 0
or
L {f 00 (t)} = s2 F (s) − sf (0) − f 0 (0) (2)
In the like manner, we can show that
If f , f 0 , . . . , f (n−1) are continuous on the interval [0, ∞) and if f (n) (t) is piece-wise
continuous on the interval [0, ∞), then
L {f (n) (t)} = sn F (s) − s(n−1) f (0) − s(n−2) f 0 (0) − · · · − f (n−1) (0) (4)
where F (s) = L {f (t)}.
Example 1.
Evaluate
L f 0 (t)
with f (0) = 1
where
1
F (s) =
s−1
Solution:
L f 0 (t) = sF (s) − 1
s
L f 0 (t) =
−1
s−1
1
L f 0 (t) =
s−1
Example 2.
Evaluate
L f 00 (t) f (0) = 1, f 0 (0) = −1
with
Solution:
L f 00 (t) = s2 F (s) − sf (0) − f 0 (0)
L f 00 (t) = sF (s) − s + 1
Example 3.
Evaluate
L f 000 (t) f (0) = f 0 (0) = f 00 (0) = 0
with
Solution:
L f 000 (t) = s3 F (s) − s2 f (0) − sf 0 (0) − f 00 (0)
It is not convenient to always use the Laplace transform definition each time we wish
to find the Laplace transform of a function f (t). In this lecture we will deal with several
labor saving properties of the Laplace transform that enable us to build up a more extensive
list of transforms without having to resort to the basic definition and integration.
.
To compute the inverse of F (s − a), we must recognize F (s). The procedures are
summarized symbolically in the following manner,
In engineering, one frequently encounters functions that are either “off” or “on.” For
example, an external force acting on a mechanical system or a voltage impressed on a circuit
can be turned off after a period of time. It is convenient, then, to define a special function
that is the number 0 (off) up to a certain time t > a and then the number 1 (on) after that
time. This function is called the unit step function or the Heaviside function, named after
the English mathematician Oliver Heaviside (1850–1925).
The inverse form of (3) is written as, if f (t) = L −1 {F (s)}, for a > 0, then
Derivatives of Transforms
If f (t) and g(t) are piece-wise continuous [0, ∞), then the product, denoted as f (t)∗g(t),
is defined by the integral Z t
f (t) ∗ g(t) = f (τ )g(t − τ )dτ (7)
0
is called the convolution of f and g.
If f (t) and g(t) with F (s) and G(s) are their respective Laplace transform, are piece-
wise continuous [0, ∞), then
The convolution theorem is sometimes useful in finding the inverse Laplace transform of the
product of two Laplace transforms.
1
When g(t) = 1 and L {g(t)} = G(s) = , the convolution theorem implies that
s
Z ∞
F (s)
L f (τ )dτ = (10)
0 s
The inverse form of (10), Z ∞
F (s)
L = f (τ )dτ (11)
s 0
Transform of an Integral
Solution:
2s + 5 2 11
L −1
=L −1
+
(s − 3)2 s − 3 (s − 3)2
2s + 5 1 1
L −1
= 2L −1
+ 11L −1
(s − 3)2 s−3 (s − 3)2
2s + 5 −1 1 1
L −1 −1
= 2L + 11L
(s − 3)2 s s→s−3 s2 s→s−3
2s + 5
L −1 = 2e3t u(t) + 11te3t u(t)
(s − 3)2
Example 4. First Shifting Theorem
Evaluate
s/2 + 5/3
L −1
s2 + 4s + 6
using the first shifting theorem.
Solution: We have,
Now,
s/2 + 5/3 −1 1 s+2 2 1
L −1
=L +
s2 + 4s + 6 2 (s + 2)2 + 2 3 (s + 2)2 + 2
s/2 + 5/3 1 −1 s+2 2 −1 1
L −1
= L + L
s2 + 4s + 6 2 (s + 2)2 + 2 3 (s + 2)2 + 2
√
s/2 + 5/3 1 −1 s 2 2
L −1
= L + √ L −1
√ √
s2 + 4s + 6 2 s2 + ( 2)2 s→s+2 3 2 s2 + ( 2)2 s→s+2
√
√ √
s/2 + 5/3 1 2 −2t
L −1 2 = e−2t cos 2 t u(t) + e sin 2 t u(t)
s + 4s + 6 2 3
Evaluate F (s)
f (t) = 2u(t) − 3u(t − 2) + u(t − 3)
Solution:
F (s) = L {f (t)}
F (s) = L {2u(t) − 3u(t − 2) + u(t − 3)}
F (s) = 2L {u(t)} − 3L {u(t − 2)} + L {u(t − 3)}
2 e−2s e−3s
F (s) = − 3 +
s s s
Solution:
L {cos t u(t − π)} = e−πs L {cos(t + π)}
L {cos t u(t − π)} = −e−πs L {cos t}
s
L {cos t u(t − π)} = − 2 e−πs
s +1
Example 9. Derivative of Transforms
Solution:
d
L {te3t } = (−1) L {e3 t}
ds
d 1
L {te } = (−1)
3t
ds s − 3
1
L {te3t } =
s−3
Solution:
d
L {t sin kt} = (−1) L {sin kt}
ds
d k
L {t sin kt} = (−1)
ds s2 + k 2
2ks
L {t sin kt} = 2
(s + k 2 )2
F (s) = L {f (t)}
Z t
F (s) = L 3t − e −
2 −t t−τ
f (τ )e dτ
0
2! 1 1
F (s) = 3 · 3 − − F (s) ·
s s+1 s−1
6 6 1 2
F (s) = 3 − 4 + −
s s s s+1
6 6 1 2
f (t) = L −1
− + −
s3 s4 s s + 1
f (t) = (3t2 − t3 + 1 − 2e−t )u(t)
Solution:
G(s) = L {g(t)}
Z 2
1
G(s) = e−st g(t)dt
1 − e−2s 0
Z 1 Z 1
1 −st −st
G(s) = e · 1 · dt + e · 0 · dt
1 − e−2s 0 0
1 1 − e−s
G(s) = ·
1 − e−2s s
1
G(s) =
s(1 + e−s )
Initial Value Problems
The Laplace transform ideally suited for solving linear initial-value problems in which
the differential equation has constant coefficients. Such a differential equation is simply a
linear combination of terms y, y 0 , y 00 , · · · , y (n) . The initial value problem
subject to
y(0) = y0 , y 0 (0) = y1 , , · · · , y (n−1) (0) = yn−1
where the coefficients and the initial values are constants can be conveniently solved by the
Laplace transform. By the linearity property the Laplace transform of this linear combination
is a linear combination of Laplace transforms
Then we have,
where L {y(t)} = Y (s) and L {g(t)} = G(s). In other words, The Laplace transform of a
linear differential equation with constant coefficient becomes an algebraic equation in Y (s).
We can write the equation in the form
Q(s) G(s)
Y (s) = + , (3)
P (s) P (s)
y(t) = L −1 {Y (s)}
The next examples illustrates the foregoing method of solving differential equations
using the Laplace transform.
Example 1.
Solution:
Solution:
2
L {x (t) + x(t)} = 4 · L
00
s + 22
2
2 0 2
s f (s) − f (0) − sf (0) + f (s) = 6 2
s + 22
12
s2 f (s) − 1 − s(3) + f (3) = 2
s + 22
12
(s2 + 1)f (s) = 2 + 3s + 1
s +4
12 + 3s3 + 12s + s2 + 4
(s2 + 1)f (s) =
s2 + 4
3s + 12s + s2 + 16
3
(s2 + 1)f (s) =
s2 + 4
3s + 12s + s2 + 16
3
1
f (s) = 2
· 2
s +4 s +1
As + B Cs + D
f (s) = 2 + 2
s +4 s +1
Using partial fraction decomposition to solve for A, B, C, D we have A = 0, B = −4, C = −4
and D = 5
−4 −4x + 5
f (s) = + 2
s2
+ 4 s + 1
2 s 1
L {f (s)} = −2L
−1 −1
+ 3L −1
+ 5L −1
s2 + 4 s2 + 1 s2 + 1
x(t) = −2 sin 2t + 3 cos t + 5 sin t
Example 3.
Solution:
Solution:
[Boyce and Diprima, 2001] Boyce, W. and Diprima, R. (2001). Elementary Differential
Equations with Boundary Value Problems. John Wylie & Sons, Inc.
[Bronson and Costa, 2009] Bronson, R. and Costa, G. (2009). Schaum’s Outline of
Differential Equations. McGraw-Hill Education, 3rd edition.
[Lebl, 2019] Lebl, J. (2019). Notes on Diffy Qs: Differential Equations for Engineers.
Independently Published.
[Rainville et al., 2013] Rainville, D., Bedient, P., and Bedient, R. (2013). Elementary
Differential Equations: Pearson New International Edition. Pearson Education, Limited.
[Zill, 2012] Zill, D. (2012). A First Course in Differential Equations with Modeling
Applications. Cengage Learning.